lsat_pt 49_expl_web

50
LSAT * PrepTest 49 Explained

Upload: nipunscribd

Post on 15-Nov-2014

125 views

Category:

Documents


1 download

TRANSCRIPT

Page 1: LSAT_PT 49_Expl_web

LSAT*

PrepTest 49Explained

Page 2: LSAT_PT 49_Expl_web

© 2006 Kaplan, Inc.

All rights reserved. No part of this book may be reproduced in any form, by photostat, microfilm, xerography, or any othermeans, or incorporated into any information retrieval system, electronic or mechanical, without the written permission ofKaplan, Inc. LSAT is a registered trademark of the Law School Admission Council.

Page 3: LSAT_PT 49_Expl_web

SECTION ILOGIC GAMES

Game 1: Twelve Films, Six Days

Situation: An “international film retrospective.”

Entities: Twelve films, six days.

Action: Six films are shown, one on each day. So we’llhave to select the six films, then put them in order—aHybrid Selection/Sequencing game.

Limitations: The six days are consecutive. Exactly sixof the twelve films are shown (so exactly six of themare not shown). One film is shown on each day. Thereare two films in each of the languages used.

The Initial Setup: We’ll start off with our basicSequencing sketch, numbering the days 1 through 6. Itwill be easy to combine a Selection sketch with thisone; in fact, selection is the easiest action to add to aHybrid sketch, since a Selection setup is just a rosterof entities, and we write out a roster of the entities nomatter what action we’ll be doing in the game. Here, wejust have to remember that we’ll be circling some ofthe films and crossing others out. Also, remember thatwe should write the letter for each type of film twice,since we have two of them available:

The Rules:

Rule 1 rules out showing a Norwegian film on two ofthe days, day 2 and day 4. We’ll just jot down “no N”beneath those days:

Rule 2 is simple, as long as you remember that “No Aunless B” means “If A, then B.” So this rule wouldtranslate as, “If a film in Italian is shown, then a film in

Norwegian is shown the next day:”

Don’t forget to formulate the contrapositive. Here, itwould be, “If a film in Norwegian is not shown the dayafter a film in Italian, then the film in Italian is notshown.”

Notice how specific our contrapositive is. Manystudents would write the trigger of this contrapositiveas “If not N,” but that would suggest that the rule isonly triggered when a film in Norwegian isn’t shown atall, and that doesn’t tell the whole story. Don’t fall intothe trap of changing difficult Formal Logic statementsto make them easier.

Rule 3 follows the same formulation as Rule 2, only ituses Greek and Italian:

Deductions: There’s one quick and obvious duplicationthat allows us to combine Rules 2 and 3 to make a bigBloc of Entities. If a film in Greek is shown, then a filmin Italian is shown on the next day and a film inNorwegian is shown the day after that:

The contrapositive is a bit more complicated here, butit still follows the same rules as all Formal Logicproblems. Just make sure you’re clear that the filmfestival could show an Italian film or a Norwegian filmwithout also showing a Greek film.

There’s one more duplication in this game, and it tellsus more about where that Bloc of Entities can fit. A filmin Norwegian cannot be shown on day 2 or day 4, so afilm in Italian cannot be shown on day 1 or day 3, sinceRule 2 would force the Norwegian film onto the nextday. Furthermore, a film in Greek cannot be shown onday 2, day 5, or day 6 for the same reason. We can fillthis information into the slots of our sketch just like wedid the first rule:

G I NIf G →

G I N → not GIf not

G IIf G →

G I → not GIf not

I N → not IIf not

I NIf I →

1 2 3 4 5 6

no N no N

FF GG HH II NN TT

1 2 3 4 5 6

Section I: Logic Games

1

Page 4: LSAT_PT 49_Expl_web

The Final Visualization:

This game may have been a Hybrid game, but the setupwasn’t much of a challenge. If you were brave enoughto attempt it right away, the payoff came in thequestion set. There’s a total of seven points availablein this game, and none of them should have posed toomuch difficulty. Let’s look at them now.

The Questions:

1. (E) Acceptability

Use the rules to eliminate wrong choices inAcceptability questions.

Rule 1 eliminates (B), which has a Norwegian film onday 4. Rule 2 eliminates three choices, (A), (C), and(D); both (A) and (D) have a film in Italian that isfollowed by a film in something other than Norwegian,and in (C), the film in Italian is shown on day 6, so itis followed by nothing at all! Only (E) remains, so itmust be the correct answer.

2. (D) “could be true EXCEPT” / “If” clause

Questions in Hybrid games will usually force you tolook at both of the game’s actions.

Take a moment to characterize the choices in thisquestion. The four wrong answer choices are all filmsthat could be shown on day 1, while the correct answeris a film that cannot be shown on day 1. The “if” in thisquestion gives us some pretty concrete information:both of the films in Italian are shown, and they’reshown on day 2 and day 5. Rule 2 tells us that a filmin Italian must be followed by a film in Norwegian, sowe can plug this information into our new sketch.Remember, we’ll also circle each of the films as it isselected:

From this, we can quickly see that both of the films inNorwegian are already selected, so a film in Norwegiancan’t be the film shown on day 1. (D) is the correctanswer. And as it turns out, this question really onlytested selection after all.

(A), (C), (E) The films in French, Hungarian, andTurkish are all floaters. They can be shown on any givenday, as long as that day isn’t already taken.

(B) The film in Greek could be shown on day 1. In fact,that would give us the “G I N” Bloc of Entities from ourDeductions.

3. (A) “must be false” / “if” clause

Seek out questions where your previous work makesit easier to find the correct answer.

Once again, both of the films in Italian are shown,which means that both of the films in Norwegian will beshown immediately after them. But this questiondoesn’t give us specific days on which to show theItalian films—they could go anywhere, as long as theydon’t violate another rule. A quick look at our MasterSketch will tell us where we can’t put the Italian films—day 1 and day 3. And a look back at question 2 tells usthat we could put the Italian films on day 2 and day 5.So is there anywhere else they could go, or are thefilms locked in to those possibilities? Well, it’s a littlebit of both. The first Italian film has to be shown on day2. It can’t be shown on day 1 or 3, and if it’s shown on

FF GG HH II NN TT I N I N 1 2 3 4 5 6

FF GG HH II NN TT

1 2 3 4 5 6 no I no N no I no N no G no G no G

I NIf I →

I N → not IIf not

G IIf G →

G I → not GIf not

G I N → not GIf not

G I NIf G →

1 2 3 4 5 6 no I no N no I no N no G no G no G

PrepTest 49 Explained

2

Page 5: LSAT_PT 49_Expl_web

day 4, then there won’t be room for a second Italianfilm later. But the second Italian film could be shown onday 5, as it was in the last question, or it could beshown on day 4. Our sketch from the last questiongives us one possibility, and we can quickly jot downthe other:

We can use both of these sketches to scan the choicesfor something that must be false. In doing so, we find(A): day 3 is always occupied by a Norwegian film, soit cannot also show a French film.

(B) A Greek film could be shown on day 1; in fact, wesaw this when we eliminated (B) from Question 2.

(C) If the second Italian film is shown on day 4, then aHungarian film could be shown on day 6.

(D) Our sketch for this question shows a Norwegianfilm on day 5, so this one definitely could be true.

(E) A Turkish film could be shown on day 4 as long asthe second Italian film is shown on day 5.

4. (B) Complete and Accurate List

Your work on previous questions can help you answerComplete and Accurate List questions.

The LSAT practically gives us a free point on thisquestion, as long as we’re paying attention to what’scome before. The last two questions have dealt withthe Italian films, and we’ve already figured out that theycan be shown on day 2, day 4, and day 5, which is (B).In fact, we could have eliminated every choice except(B) and (C) just by using the “if” clause from question2. Always check your work from previous questions—itcan save you a lot of time in situations like this.

(A) and (D) include days 1 and 3. We figured out duringthe Deductions that the Italian film can’t be shown onthose days, because it would force the Norwegian filmto violate Rule 1.

(C) comes close, but it skips day 4 and puts the Italianfilm on day 6. In this case, we couldn’t show theNorwegian film on the day after the Italian film, sincethere aren’t any more days.

(E) is an accurate list, but it isn’t complete—the Italianfilm could also be shown on day 5.

5. (D) “could be true” / “if” clause

Use all of the rules to make as many deductions aspossible in your new sketch before moving on to theanswer choices.

The correct answer in this question must be veryspecific: a film that can be shown on day 1, followed bya film that can be shown on day 6. The four wrongchoices will include a film that can’t be shown on day1 in the first slot, or a film that can’t be shown on day6 in the second slot, or both.

The new “if” in this question might not make youimmediately leap to any new deductions, but that’s justfine. You should still follow the steps of the KaplanMethod. Recopy your sketch, plug in the new rule, thensee if you can make any deductions.

Once again, the work we’ve done in previous questionscan help us out here. It may not be immediatelyapparent, but if you run through Rules 2 and 3 andthink about where you could place the Blocs of Entities,you’ll realize that there just isn’t room for them. Theonly place where there are two consecutive daysavailable is day 1 and day 2, and since a Norwegianfilm can’t be shown on day 2, we can’t show an Italianfilm on day 1. This means that we’ve triggered thecontrapositives of Rules 2 and 3, so we can plug in theresults as well—we won’t select an Italian film, and wewon’t select a Greek film. Go ahead and cross them offof our list of entities:

Now we’re ready to check the choices. Let’s go throughthem together:

(A) must be false. Both of the French films are alreadyplaced, so one of them cannot be shown on day 1.Eliminate.

(B) must be false. A Greek film cannot be shown on thefirst day. Eliminate.

(C) must be false. An Italian film cannot be shown onday 1. Eliminate.

(D) could be true, and is correct. Be careful not toreverse Rule 2—a Norwegian film can be shown

FF GG HH II NN TT

FF GG HH II NN TT F F 1 2 3 4 5 6

FF GG HH II NN TT I N I N 1 2 3 4 5 6

Section I: Logic Games

3

Page 6: LSAT_PT 49_Expl_web

without an Italian film before it. For the record:

(E) must be false. A Greek film cannot be shown last.Eliminate.

6. (E) “must be true” / “if” clause

When the “if” clause in a Hybrid game deals with asingle action, work out as much as you can with thataction before taking any other steps.

The new rule in this question points us to the selectionpart of this game. We know right off the bat that we’llreject both French films and both Italian films. Youshould immediately ask yourself, what else does thatmean for our selection of entities? A quick glance atthe contrapositive of Rule 3 tells us that if we don’tshow an Italian film, we can’t show a Greek film. So wecan cross the Greek films off of our roster as well. Allof a sudden we’re done with one of our actions! We’verejected six of the films, so the remaining six must beselected:

Now we just have to sequence those six entities. We’vealready used Rules 2 and 3, so only Rule 1 is left—days 2 and 4 cannot be the days on which theNorwegian film is shown, so each one of those daysmust show either a Hungarian or a Turkish film. Jot thatdown as well:

When the correct answer is something that must betrue, the four wrong answer choices will all be thingsthat could be false. Here, the choice that must be trueis (E)—as we can see from our sketch, either aHungarian or a Turkish film must be shown on day 2.

(A) and (B) could be false. A Norwegian film could beshown on day 1 or on day 5, but it doesn’t have to be.The Norwegian films could just as easily be shown onday 3 and day 6.

(C) could be false. A Turkish film could be shown onday 4, but so could a Hungarian film.

(D) could be false. A Turkish film could also be shownon day 3.

7. (D) “must be true” / “if” clause

Deductions don’t have to nail entities down tospecific slots to be useful; sometimes, a deductionthat gives you the order of entities can be enough toanswer a question.

In Question 7, we’re looking for a day or set of days onwhich a film in Norwegian must be shown. The incorrectanswer choices will have a day or days on which someother film could be shown. But instead of giving us anydefinite positions, the “if” clause in this question onlytells us the order of two of the films: a film in Greek willbe shown after a film in Norwegian. We can’t nail thoseentities down to positions just yet, but we can write outa bit of shorthand for this new rule.

Even that small amount of information will allow us tomake a deduction or two. Rule 3 tells us that a Greekfilm must be immediately followed by an Italian film,and that Italian film must be immediately followed by aNorwegian film (Rule 2). Now we’re getting somewhere:

We still can’t be absolutely certain where any of theseentities go, but a big Bloc of Entities like “G I N” canonly fit a few places. It takes up three consecutivedays, so the absolute latest it could fall is on days 4,5, and 6. That would only leave the first three daysopen for the Norwegian film which must come beforethe Greek film, and Rule 1 tells us that we have toeliminate one of those days from contention: theNorwegian film cannot be shown on day 2. That leavesus only two days on which the Norwegian film can beshown: day 1 or else day 3. This means that (D) mustbe true.

(A) The Norwegian film could be shown on day 1, butit also could be shown on day 3. Eliminate.

(B) The Norwegian film could be shown on day 3, butit could also be shown on day 1. Eliminate.

(C) The second Norwegian film could be shown on day5, but it could also be shown on day 6. Eliminate.

(E) The first Norwegian film could be shown on day 1while the second Norwegian film is shown on day 6.Eliminate.

N … G I N

N … G

H/T H/T 1 2 3 4 5 6

FF GG HH II NN TT

PrepTest 49 Explained

4

Page 7: LSAT_PT 49_Expl_web

Game 2: Housemates and Mail

Situation: A house or apartment shared by three women.

Entities: The women, and their five pieces of mail.

Action: To assign the pieces of mail to the women. Twosets of entities make this a Matching game, but itwouldn’t hurt (or change your sketch) if you saw it asDistribution.

Limitations: There are five pieces of mail among threeaddressees, so someone’s going to get more than onepiece of mail. It’s also important to note (sentence 3of the overview paragraph) that each housemate getsat least one piece of mail...

The Initial Setup: ...so our sketch should first andforemost indicate the minimum single slot, under eachhousemate’s name, for the minimum one piece of mailshe receives, and a reminder that two slots are yet tobe placed. (And for clarity’s sake we’ll spell out thenames to differentiate them from the mail.)

In game after game, job one is to get the slots divviedup. Then, and only then, should you be concerned withwhich entity fills each slot.

The Rules:

Rule 1, a negative rule, needs to turned into thepositive. If Georgette gets neither letter nor magazine,then of course she will get one of the other pieces—the flyer, the postcard, or the survey. But the bigrealization here is that both letter and magazine willhave to end up in Jana or Rini’s columns. Of course wedon’t exactly know who gets which, and may not do sountil the questions come along. For now, arrows from“l” and “m” to the Jana and Rini columns should be anadequate reminder.

Rule 2 is a classic if/then rule that, as always, needsto be understood in its contrapositive form as well.Whenever we place the letter in Rini’s mailbox, Janawill receive the postcard. It follows, then, that should

the postcard go to a housemate other than Jana, theletter will go to someone other than Rini. Think thisthrough and jot it down carefully; don’t rush. We don’tneed to memorize the rule, but we do need to have itproperly noted for reference during the questions:

Rule 3 is a little vague, but which of these rules is not?We need to remember that we’ll never see “f” alone ina column; that at least one and perhaps two otherpieces of mail will appear there as well. A little sketchshowing f’s slot above at least one other slot shouldsuffice.

Deductions: We’re going to recommend the setting upof Limited Options in this game—not so much becausedoing so is required (it’s never required), but becausethe setup will speed us through the questions. And thelimited options are based on the Duplication (that’s theD in BLEND) of the letter in Rules 1 and 2.

Since Georgette doesn’t get the letter, then either Janaor Rini does. There are your two options, based onwhich housemate receives the letter. And since in thelatter case, Rini’s getting the letter assigns thepostcard to Jana, the options really look like so:

We can therefore see that, at minimum, Jana receivesthe letter or the postcard. There’s no other way. (By theway, note that in Option I, the postcard can go to anyof the housemates, because Rule 2 no longer applieswhen Rini doesn’t get the letter.)

What about Georgette? Well, this is interesting. InOption I, Georgette will get at least one of the threepieces of mail Rule 1 leaves open to her: flyer,postcard, survey. And we must keep in mind that if shegets the flyer, she’s got to get at least one of theothers. Rini has, in the same way, several possibilitieshere.

But in Option II we can be much more precise. With (aswe’ve seen) Rini getting the letter and Jan the

GEO JAN RIN

l

Op. I

GEO JAN RIN

p l

Op. II

If RIN = l → JAN = pIf JAN = p → RIN = l

f l m p s

GEO JAN RIN

+

Section I: Logic Games

5

Page 8: LSAT_PT 49_Expl_web

postcard, suddenly Georgette is down to only twopossibilities: flyer and survey. And since the flyercannot be delivered alone, we can be sure that inOption II, Georgette will either receive the survey only,or the survey and the flyer—either way, she definitelygets the survey. It all might look like so:

The Final Visualization:

The Questions:

8. (B) Acceptability

When checking rules against choices, don’t neglectany rules that might be buried in the openingparagraph.

First scanning for Georgette’s receiving the letter ormagazine (Rule 1), we knock out (C) where the latter isthe case. Next, we might want to check for instancesof the flyer’s appearing with nothing else, in violation ofRule 3. That yields another one to reject, (E).

In the remaining choices, we expect to find at least oneviolator of Rule 2, and we do, in (D), where Rini gets

both the letter and Jana’s postcard. But we have twochoices remaining and we’re out of rules. Or are we?

We are not. Built into the opening pargraph is the listof the five pieces of mail. In (A), no one gets apostcard at all, so that won’t work. (B) is left as theviolator of no rules and the right answer (Note that (B)is an instance of our Option I in action).

9. (B) Complete and Accurate List

Setting up Limited Options invariably yields quick andeasy points.

What can Jana’s one and only piece of mail be? Giventhat our options reveal that, at base, she must receiveeither the letter (thanks to Rule 1) or postcard (thanksto Rule 2), (B) must be correct. (A)’s list is incomplete(must include the letter) while (C)’s, (D)’s, and (E)’sare all inaccurate: Jana can never receive “magazineonly” or “survey only.”

10. (E) Complete and Accurate List /“CANNOT be true”

Never assume that each question covers new ground.There is often much repetition, even among a groupof only five questions.

Jana again. This time, four of the five choices arepossible lists of Jana’s items, and the right answer isa violator in some way. Well, if we recognize, as we justdid in question 9, that at minimum, Jana’s list mustcontain the letter or the postcard, we can instantlyrealize that (E), which mentions neither of them, isimpossible. Of the wrong choices, (A), (B), and (C) areall possible under Option I, and (D) would work underOption II.

11. (B) Complete and Accurate List /“CANNOT be true”

Keep using your Limited Options, and re-draw onlywhen you need to.

This time, Rini is the center of attention, but otherwisethe question is worded exactly the same as theprevious one. We seek a violator, and our options canhelp us determine whether a choice will work or not.

(A) If, as (A) proposes, Rini gets magazine andpostcard only, then we’re in Option I, with severalacceptable ways to distribute the mail among her otherhousemates. Acceptable. Eliminate.

(B) has Rini getting the letter and survey only. Theformer means that we’re in Option II, and the latter

GEO JAN RIN

f/p/s l Op. I

GEO JAN RIN

s p l

Op. II

or f + s

f

(min.)

m

f l m p s

→→

m→→

PrepTest 49 Explained

6

Page 9: LSAT_PT 49_Expl_web

means that (B) is the right answer, the unacceptabledistribution. As we’ve seen, in Option II Georgette mustget the survey. Of the remaining choices, (D) is anapplication of Option I, while (C) and (E) are possiblewithin Option II.

12. (E) “Could be true” / “if” clause

Turn abstract “if” clauses into concrete ones.

So the magazine and survey go to one housemate, eh?It’s our job to ask and answer the question: Which one?Not Georgette, surely, because she never gets themagazine at all. Must be Jana or Rini.

If it’s Jana—work it out now—then she must also receiveeither the letter or the postcard (that’s the basis of ourtwo Options, right?: that at minimum, she receives oneor the other). Problem: that leaves two housemates withnothing, and the flyer yet to be distributed. But lest weforget Rule 3, the flyer cannot be received by itself. Thiswon’t fly. It has to be the case that Rini is the mysteryhousemate of the question stem, the one who gets themagazine and survey. We might sketch this much:

The student who really knows her options will see thatthis cannot be Option II (in which Rini gets the letter aswell, but that would make the flyer and postcardindividual pieces going to housemates in violation of Rule3). So it’s Option I, which was the most flexible option allalong anyway. Jana receives the letter, which leaves theflyer and the postcard available. Georgette will takeeither the postcard alone, or the flyer and the postcard(since the flyer cannot be distributed by itself). Eitherway, she gets the postcard. Now the flyer can floataround, since everybody already has some other piece ofmail. Armed with this sketch, let’s see what the choiceshave in store.

(A) As we see, the survey goes to Rini, not Georgette.We saw that (A) is impossible right away.

(B) No, Rini cannot take up the postcard as well,because again, that would leave the flyer a solo piece forsomeone. Eliminate.

(C) Nope, the magazine goes to Rini, as we’ve alreadyset up. Eliminate.

(D) Nope, again, this is Option I, where Jana gets theletter. Eliminate.

(E) is left over and must be correct. Indeed, Jana couldget the flyer, like so: Georgette = postcard; Jana = flyerand letter; Rini = magazine and survey. (E) is correct.(For the record, another proper matchup would be:Georgette = flyer and postcard; Jana = letter; Rini =magazine and survey.)

GEO JAN RIN

? l/p m

s

Section I: Logic Games

7

Page 10: LSAT_PT 49_Expl_web

Game 3: Summer Courses

Situation: A summer program.

Entities: The courses being offered: geography,history, literature, mathematics, psychology, sociology,and zoology.

Action: The program will offer “at least one” of thecourses. We’ve got to figure out which course orcourses are offered, which makes this a Groupinggame of Selection.

Limitations: “At least one” is our only limitation. Thatdoesn’t really give us a lot of information, which is aclue that we’re probably dealing with a harder game.

The Initial Setup: For all Selection games, our basicMaster Sketch is a roster of the entities:

We’ll circle or cross these entities out as we learnmore about which are selected or rejected.

The Rules:

All of the rules for this game are in formal logic.Coupled with the fact that we don’t have any numericallimitations to help us figure out the game, this tells usthat this game is almost certainly the hardest one inthe section. Rule 1 starts us off with a formulationwe’re not used to seeing: if mathematics is offered,then either literature or sociology, but not both, isoffered. Even as you translate this rule into algebraand form the contrapositive, keep it in a completesentence in your mind—that will help quite a bit. Thisrule would translate like so:

Think through the contrapositive for this rule. To negatethe original rule, we would have to say, “If the programoffers neither literature nor sociology, or if it offersboth literature and sociology, then it will not offermathematics.” Our contrapositive looks like this:

As is often the case, it’s much easier to think throughthis rule as a complete sentence than it is to dealexclusively in algebra.

Rule 2 is much more straightforward—we’ve seenrules similar to this one in our practice. We cantranslate this rule like so:

The contrapositive is also fairly straightforward:

Rule 3 has exactly the same formulation as Rule 2,just using different entities. Its translation andcontrapositive are as follows:

Finally, Rule 4 gives us the simplest Formal Logicformulation of the entire game. If geography is offered,then both history and zoology are offered, meaningthat (in the contrapositive) if either history or zoology isnot offered, then geography is not offered.

Deductions: We’ve already made several deductions inthe form of working out the contrapositives of eachrule. Unfortunately, the nature of the rules in this gamemakes it almost impossible to deduce any further. Wecan draw a bit more information out of Rules 2 and 3:remember from our Formal Logic work back in Lesson2 that the formulation “If A, then not B” can also bewritten as “Never AB.” So from the rule that if literatureis offered, then psychology is not, we can note “NeverLP,” and from the rule that if sociology is offered, thenzoology is not, we can note “Never SZ.” Those may notseem like much, but any deduction will help us answerthe questions.

We could probably work out a few more duplicationsbetween the Formal Logic rules, but it’s really notworth the time involved. We’ll need to check every ruleof the game against the “if” rules in each question, sowe’ll be able to catch all of the duplications as we go.Working through them now would be a waste of effort.

If G → H and ZIf not H or not Z → not G

If S → P and not ZIf Z or not P → S

If P or not G → not L

If L → G and not P

If LS or not L and not S → not M

If M → L or S and not LS

G H L M P S Z

PrepTest 49 Explained

8

Page 11: LSAT_PT 49_Expl_web

The Final Visualization:

The Questions:

13. (A) Acceptability

A Complete and Accurate List of the entities selectedis really an Acceptability question.

The first question of this game forces us to check eachof the selections in the answer choices. But as with allAcceptability questions in Selection games, we won’treally be able to tell whether an answer choice is rightor wrong unless we also look at what is not listed—theentities that are rejected. Once we recognize that we’llhave to do that, we can simply use the rules toeliminate choices like we do in every otherAcceptability question.

(E) violates Rule 1, since the summer program cannotoffer mathematics along with both literature andsociology. (C) also violates this rule, sincemathematics cannot be offered without eitherliterature or sociology. (D) violates Rule 2, sinceliterature and psychology cannot both be offered. Rule3 doesn’t help us, but Rule 4 eliminates (B): ifgeography is offered, then both history and zoology arealso offered. (A) remains, and is correct.

14. (C) “could be true” / “if” clause

Remember to check the implications of every rulebefore going to the answer choices.

As always with “if” questions, we’ll write out a copy ofour Master Sketch, plug in the new rule, then test it for

any deductions. Our initial setup for the question willlook like this:

Now we can check through the rules. “If L” is a triggerin Rule 2, and it tells us that geography is offered, butpsychology is not. We can circle G and cross out P:

Let’s see if this new information triggers any morerules. Sure enough, “If G” means history and zoologyare offered (Rule 4), and “Not P” means sociology willnot be offered (Rule 3’s contrapositive).

A quick look at Rule 1 shows that we’re done with ourdeductions, since we can’t be certain of anything else.But we’ve come quite a long way. There’s only oneentity left that we aren’t certain about: mathematics.The math course could be offered, or it could not—either way, it won’t violate a rule. This means that (C)is the correct answer, since mathematics could indeednot be offered.

(A) Our deduction from Rule 3 told us that sociologycannot be offered. (A) must be false.

(B) and (E) Rule 4 told us that both history and zoologywere offered as soon as we realized that we had tooffer geography. Both of these choices must be false.

(D) Count the number of courses offered: in thisscenario, there must be either four or five. (D) must befalse.

15. (A) “CANNOT be true” / “if” clause

Don’t let negatives in the question stem throw you offtrack.

Trying to go too fast causes a lot of errors on the LSAT.For example, if you read this question stem as, “Ifhistory is offered by the summer program,” then youalmost certainly got the wrong answer. Here, we’relooking at what happens if history is not offered. Thecorrect answer will be another course that cannot beoffered in the summer program. The four wrong choiceswill all be courses that could or must be offered by theprogram. As usual, we’ll draw out a sketch and workthrough the rules to find the correct answer.

G H L M P S Z

G H L M P S Z

G H L M P S ZIf M → L or S and not LS

If LS or not L and not S → not M

If L → G and not PIf P or not G → not L

If S → P and not ZIf Z or not P → S

If G → H and ZIf not H or not Z → not G

Never LP Never SZ

G H L M P S Z

Section I: Logic Games

9

Page 12: LSAT_PT 49_Expl_web

First of all, “Not H” triggers the contrapositive of Rule4, so we know that geography is not offered.Unfortunately, geography isn’t one of the choices, sowe’ll have to keep going. “Not G” triggers thecontrapositive of Rule 2—if geography isn’t offered,then literature is not offered either. Now we’re gettingsomewhere:

Literature is in the answer choices, in (A). We’ve foundthe other course that cannot be offered, so we’vefound our correct answer.

(B) Selecting mathematics would require that we alsoselect either literature or sociology. Literature cannotbe selected, but sociology can, so mathematics can beselected.

(C) Psychology can be selected without requiring anyother courses to be offered.

(D) If sociology is offered, then psychology must alsobe offered (Rule 3), but this would not violate anyrules.

(E) If zoology is offered, then sociology is not offered(Rule 3), which means that mathematics is also notoffered (Rule 1). All of that would be just fine.

16. (D) “must be true” / “if” clause

Don’t be afraid to draw out more than one sketch fora question if it becomes necessary.

The “if” rule of this question gives us a morecomplicated situation than we’ve seen in the rest ofthe game. If mathematics is selected, then eitherliterature or sociology (but not both) must also beselected. We can’t be absolutely certain of which onewould be selected, and this is a situation where mosttakers of the LSAT would be stumped, wondering whereto go. Instead of worrying about which of the twooptions we must select, we’ll simply draw out and testeach one—think of this as a miniature Limited Optionssketch, for this question alone. One of our sketcheswill show what happens when mathematics andliterature (but not sociology) are selected, and one ofthem will show what happens when mathematics andsociology (but not literature) are selected:

Let’s apply each of the rules to these sketches inorder. Rule 2 will help us with Option I, and Rule 3 willhelp us with Option II. If literature is offered, thengeography is also offered but psychology is not. On theother hand, if sociology is offered, then psychology isoffered but zoology is not. The next step in eachoption, filling in the implications of these two rules,looks like this:

Rule 4 is the only rule we haven’t applied, and it worksin both options. If geography is offered, as in Option I,then history and zoology are also offered. But in OptionII, zoology is not offered, so the contrapositive of Rule4 tells us that geography cannot be offered in thisoption. Now we can finish each sketch:

Armed with all this information, the answer choices area snap. The correct answer must be true, which meansit is true in every situation. For this question, thatmeans it will be true in both of our options, while anychoice that could be false in either option is wrong. Theonly answer choice that is true in both options is (D):at least three courses must be offered.

(A) and (C) were easy to eliminate immediately: eitherliterature or sociology is offered, but neither one ofthem must be offered.

(B) Psychology is definitely offered in Option II, but itcannot be offered in Option I. Eliminate this choice.

Option I: G H L M P S Z

Option II: G H L M P S Z

Option I:G H L M P S Z

Option II:G H L M P S Z

Option I:G H L M P S Z

Option II:G H L M P S Z

G H L M P S Z

G H L M P S Z

PrepTest 49 Explained

10

Page 13: LSAT_PT 49_Expl_web

(E) Five of the courses are offered in Option I, whichmeans this choice could be false. However, anyonewho only looked at what happens when sociology isselected would have been fooled, since Option II offersat most four courses.

17. (E) “must be false”

A “must be false” question at the end of a questionset is a perfect place to use your previous work.

The correct answer to this question is something thatmust be false, so the four wrong choices will all bepossibilities, things that could be true in some situation.Well, at this point in the game, we’ve seen severalpossibilities—in fact, we’ve seen at least one in everyquestion that starts with “if,” as well as the possibilityin the correct answer to the Acceptability question. Thatwork we’ve already done will save us time workingthrough the answer choices for question 17.

(B), (C), and (D) all list possibilities that includemathematics. It’s a good thing we just finished workingthrough question 16, which includes both of theoptions that occur when mathematics is offered. Thoseoptions allow us to eliminate all three of these choices.(B) and (D) are eliminated by Option I, which showshow mathematics can be offered along with bothgeography and history, and (C) is eliminated by OptionII, which shows mathematics offered with psychology.

That leaves (A) and (E), but unfortunately, neither ofthese choices lists a possibility that appeared in ourprevious work. We’ll have to draw out a sketch to testthese choices. However, we can save ourselves a bit oftime by finding a way to test both of these choices atonce—here, both (A) and (E) include geography alongwith another course. If we see what happens whengeography is selected, that should tell us which of theother courses cannot be offered in tandem.

Rule 4 tells us that when geography is offered, bothhistory and zoology are also offered. There’s wherewe’ll start our sketch.

A quick skim of the rest of the rules leads us to thecontrapositive of Rule 3: if zoology is selected, thensociology is not selected. Cross sociology off of thesketch:

“Not S” doesn’t lead us to any further deductions, butthat’s just fine. We’ve managed to find the correctanswer. Geography and sociology cannot be selectedtogether, so (E) must be false.

(A) Both geography and psychology could be selected.Rule 2 might have confused the issue with this choice,since it involves both geography and psychology, butthey are only mutually exclusive when literature isselected. As long as literature is rejected, these twocourses can both be offered.

(B), (C), and (D) We saw all of these possibilities inquestion 16. Mathematics can be offered along withgeography, psychology, and history—just not all ofthem at once.

G H L M P S Z

G H L M P S Z

Section I: Logic Games

11

Page 14: LSAT_PT 49_Expl_web

Game 4: Computer Processor Chips

Situation: A computer company or lab—anywherecomputer chips are assembled.

Entities: Eight chips, F through O (with I and Nmissing).

Action: Sequencing—to rank them from fastest (#1) toslowest (#8).

Limitations: According to Rule 1 there are no ties, sothis is a pretty clean sequence, eight slots for eightentities.

The Initial Setup: We can work either horizontally orvertically; choosing the former gives us:

The Rules:

Rule 2 is one of those we welcome, because we canput them right into the sketch: “F/G” will take slot 1,leaving one of them, F or G, to go elsewhere.

Rule 3, a negative rule, doesn’t take us too far.Somehow we have to remember never to insert M onthe far right. A notation “M � 8” might do the trick, orwe could write “No M” next to the last slot.

Rule 4 offers us a workable Bloc of Entities.Somewhere in the sequence—somewhere to the rightof slot #1, of course—we need to see “H _ J” and weshould jot that down nearby.

Rule 5 presents an even more influential bloc becauseit involves more slots: somewhere we will see“K _ _ L.” Note for the moment that those two blocs,Rule 4’s and Rule 5’s, involve seven slots betweenthem; there are only eight slots in the entire game, andone of them, #1, goes to F or G. Surely there will be amajor overlap between these two blocs.

Rule 6 introduces the last of the eight to appear in arule, and confirms that O will sit to the right of both Jand L—whom we’ve heard from already.

Deductions: BLEND reminds us to use duplicationsand blocs, and the bigger the bloc the better. Since Lis duplicated, mentioned in both our most influential

bloc (Rule 5’s), and in Rule 6, let’s build on it. Thesequence must contain:

Now consider this: what we’ve just drawn deals with sixof the eight slots. “H _ J” has to fit in there too, andmust overlap somehow with the “K and L” bloc. (Seewhy? If, for instance, we had “F/G H _ J K _ _ L O,”that would be nine slots, not eight.) One of the twoslots between K and L will have to be occupied by H orJ—too many possibilities to draw out, but we shouldsee that it’s the case.

Moreover, our sketch should reflect that “H _ J” has tofall earlier in the sequence than O, because of Rule 6.

So which chip is slowest? O is a candidate, as is one ofthe F/G pair—whichever one doesn’t take slot 1. All theother chips are prohibited from slot 8: M because of Rule3, and H, J, K, and L because the rules mandate that allof them are faster than one or more other chips.

The Final Visualization:

Why not draw out all the possibilities? First, becauseone quickly sees that there are too many to jot downquickly; second, because we only have five questions. Itwill be faster to just work through the questions than towrite out possibilities that might not even be used. Let’suse the concrete information provided, and build on it.

The Questions:

18. (B) Partial Acceptability

Learn to recognize an Acceptability question even ifthe word “acceptable,” or one of its synonyms, isn’tmentioned.

Each choice offers two assignments of chip to slot, butonly one choice will violate no rules. If you compareyour Master Sketch to the choices, you should quicklysee that Rule 3 is blatantly violated by (A): M can never

F/G ... K L ... O

1(M = 8)

H J

F/G ... K L ... O

F G H J K L M O

1 2 3 4 5 6 7 8

PrepTest 49 Explained

12

Page 15: LSAT_PT 49_Expl_web

take slot 8. Neither can L (E), because O is slower thanL according to Rule 6. So those two choices drop out.J can’t be third (C) because either F or G takes slot 1,and J is preceded by two other chips (Rule 4)—thefastest J can be is fourth. If K were second and H third(D), look at our two big blocs: that would force J and Linto the same slot! But “There are no ties” (Rule 1). That leaves (B), whose assignment must work (sinceall the other choices have been tossed); and indeed“F H K J G L M O” is a satisfactory sequence that isconsistent with (B). Set this possibility aside for use inanswering later questions.

19. (E) “CANNOT be true”

Use your Master Sketch and previous scratchwork toeliminate choices.

Where can H not rank? Well, we just saw in Question 19that H can take slot 2, so (A) can be eliminated. Thepossible overlaps of the H&J and K&L blocs allow formany possibilities, but on the right side of the sketchwe should notice that no matter what, H has to befollowed by some chip, then J, then O (“H _ J ... O.”)Which means that H cannot be any slower than fifth—Hcannot occupy slot 6, in other words, and (E) is correct.

20. (B) “could be true” / “if” clause

Combine a new “if” as if it were a new rule, for thatquestion only.

The “if” clause tells us that F will be to the right of O inthe sequence, leaving G for slot 1:

The sketch includes seven of the eight slots, and yet tobe placed are M, H, and J, the latter two of whichassume the “H _ J” configuration. But the focus is onslot 2, and here we have some solid ground. G is first,not second, so we can eliminate (A). J (D) and L (E)can never occupy slot 2 because of Rules 4 and 5respectively. So our choice is between H (B) and M (C),and only the former works, in “G H K J M L O F.” If weplaced M second, H and J would be left with adjacentslots between K and L, a violation of Rule 4.

21. (D) “could be true”

When asked for the best (or worst) possible rankingof an entity, start with the extreme choice.

We are looking for the fastest possible slot—i.e., thefurthest-left slot—available to J, given that M is faster

than (“to the left of”) J. We can do some sketching, butlet’s work primarily with the choices. “The fastestranking J could have” could be...(A) ...second, but that’s never possible. J always fallsafter H and another chip, not to mention F or G in slot1. Eliminate.(B) Third is also impossible, since “F/G...H _ “ comesbefore J always. Eliminate.(C) Fourth? Promising: that would be “F/G” in 1, and“H M J” in 2–4. Slots 5–8 are left, and...wait a minute,no good. “K _ _ L” would have to take slots 5 and 8respectively, leaving O to violate Rule 6. Eliminate.(D) Fifth? We’ll start with “F/G” in 1, as always. If J isin fifth, then H would have to be in third. That meanswe’ve got to place M in either second or fourth; if M isin fourth, there’s no place for “K _ _ L” to fit, so we’llput M in second. That forces K into fourth, and L intoseventh. O has to follow after L, so O is last, and theother “F/G” fits into sixth. That works: “F/G M H K JF/G L O.” Fifth is the fastest ranking J can take.(E) Sixth? That works: “F K M H L J O G,” for instance.Too bad we’ve already seen J take fifth. Eliminate.

22. (C) “must be true”

When a “must be true” comes with no “if”-clause,look for violators among your previous scratchwork.

(A) asserts that J can be ranked no faster than fifth,but that’s untrue if we can put J fourth or earlier in thesequence—or, as we’ve been working it, further to theleft. If your scratchwork for questions 18 or 20 is stilleasily viewable (as it should be), you can see that bothtimes we placed J in the fourth slot. (A) is untrue.Eliminate.(B) You probably haven’t seen K ranked second,unless you went on to choice (E) of question 21. Butyou can quickly jot down a possibility where that’s thecase: “F K G H L J M O,” for example. (B) is untrue.Eliminate.(C) In none of our sketches have we seen L rankedfaster than fifth. We can wait and come back to thisone, or check to see whether we can find an exceptionto it. And just a quick look back at our Master Sketchreminds us that given F/G first, and “K _ _ L,” indeedthe furthest-left slot for L is fifth, and (C) is correct. Forthe record:(D) We saw M ranked second in question 21. (D) isuntrue.(E) In both questions 20 and 21 we were able to rankO faster than eighth: seventh, as a matter of fact. (E)is untrue.

G ... K L ... O ... F1

Section I: Logic Games

13

Page 16: LSAT_PT 49_Expl_web

SECTION IILOGICAL REASONING

1. (D) Point at Issue

Always use Kaplan’s Decision Tree for Point at Issuequestions.

We can get through choices in record time if we applyKaplan’s Decision Tree to this brief colloquy betweenIlana and Gustav on the topic of Raymond Carver’swriting.

(A) Only Gustav mentions compassion; Ilana neverdoes, so we cannot be sure she even holds an opinionon Carver’s compassionate qualities. Eliminate.

(B) Ilana must believe that Carver’s work ispessimistic, since she says so; and the same point isone that Gustav concedes (“Granted....”). This is apoint of agreement between them. Eliminate.

(C), like (A), focuses on a feature mentioned only byGustav and not by Ilana. This time it’s whether Carver’sstories are humorous. Eliminate.

(D) Ilana has an opinion here: she states unequivocallythat pessimism in a story is “a sure sign of inferiorwriting.” Gustav has an opinion too, but a diametricallyopposed one: in Carver, one finds a “fine” writer whosework contains pessimism. This is the point at issue.(D) is correct. For the record:

(E) Neither Ilana nor Gustav mentions “aestheticvalue,” so we cannot pin down either one’s opinion onthis term.

2. (E) Paradox

Always try to predict the kind of statement thatwould allow two apparently contradictory phenomenato coexist.

It does seem odd that last year saw both anunprecedented drop in the violent crime rate and ahuge increase in people’s worry about violent crime.You’d think they’d be reassured. There must be someexternal factor that jacks up people’s fears, and (E)provides it: the press. Heavy coverage of violent crimeswould certainly explain why people are more fearfulthan the reduced crime rate should make them.

(A) doesn’t speak at all of public reaction to crime, andof course the right answer must do so, since it’s thepublic’s anxiety that is so paradoxical.

(B) The reporting of public anxiety is after the fact:we’re concerned with what raised that anxiety in the

first place, not the way in which the anxiety wasreported afterwards.

(C), if true, deepens the paradox. These so-calledrealistic assessors of the risk of victimhood should beless, not more, anxious, given the statistics cited in thestimulus.

(D) correlates high anxiety with high crime areas, whichbegs the question of what became of violent crime inthose areas last year. We don’t know, though theshrewd guess would be that it dropped there just as itdid generally. In any event, the paradox deals with anoverall increase in public anxiety vs. an overall declinein violent crime, so focusing exclusively on the behaviorin high-crime pockets, as (D) does, is off the point.

3. (C) Principle

When matching a general precept with a specificcase, search for the answer that is as close to a 1:1matchup as possible.

The word “principle” isn’t used in the stem, but we doget a generalized definition of efficiency followed by therequest to identify a concrete example of same, andthat’s exactly how classic principle questions work.Efficiency is herein defined as the tendency to favor thebig-risk, big-return project over the low-risk, low-returnone. Let’s look through the choices as the test takerwould.

(A) A tight-deadline project is specifically one that(sentence 1) inefficient employees tend to churn out.Eliminate.

(B) has the employee eschewing a tight-deadlinereport. Sounds good, until we see that he’s going towork instead on “routine,” easily delayed stuff.Where’s the potential big return in that? Eliminate.

(C) To be punctual at a routine sales meeting would bethe norm. To skip that in favor of an “urgent” phone callfrom a “major” client offers a big risk (maybe the clientis upset; maybe I won’t handle her properly) as well asa big possible payoff (maybe she’s calling with a hugeorder). (C) has to be the answer, since it matches upwith the definition in the stimulus. Let’s make sure theremaining ones are as flawed as they need to be:

(D) A daily confab to jaw over schedules and workloadssure doesn’t sound like taking a big risk, does it? Nogood.

(E)’s employee is a slave to deadlines and sounds likea real drone as defined in sentence 1. Nope, (C) is thewinner, as we thought.

PrepTest 49 Explained

14

Page 17: LSAT_PT 49_Expl_web

4. (B) Strengthen the Argument / EXCEPT

If four choices strengthen an argument, then thecredited choice either weakens the argument or fallsoutside of its scope.

There’s a common logical flaw known in rhetoricalcircles as the post hoc argument, from the Latin posthoc ergo propter hoc or “after this, therefore becauseof this.” You needn’t know the term, but the flaw isworth remembering—it’s the assumption that becauseevent B follows event A, it must be true that B wascaused by A. Here, “A” is the young kids’ playing ofviolent video games, and “B” is the same kids’aggressive behavior. The author does in fact concludethat A leads to B, although there’s an assumption inthis argument that’s so subtle that it’s easy to miss.The author concludes that violent games leads kids tobelieve that aggressive behavior is acceptable—hemust assume that if they believe it’s okay, then they’remore likely to behave aggressively (as they did).

As we say, this is a famous reasoning flaw, but the factthat this is an “all strengthen EXCEPT” means that fourof the choices lend credence to it; and of course themore an argument is strengthened, the less flawed itbecomes. You can either look for choices that causallycement post-gaming aggressiveness and belief inaggressiveness to prior violent gaming, or simply seekthe outside-the-scope or weakener choice.

(A) offers the kids’ acceptance of one type ofaggressive behavior (i.e., the behavior of others) thatfollows violent gaming—in other words, one more pieceof evidence that links the two phenomena. Eliminate,because this is a strengthener.

(B) If kids who have never played violent games findviolence acceptable, then that offers some indicationthat the two phenomena are independent: perhapsthey’re not causally linked at all. (B) must be thecredited choice because it’s a 180: it tends to weakenthe alleged causality. Let’s see how the remainingones strengthen, just to build our confidence and skill:

(C) offers the inverse of the logic. If non-violent gamescause no increase in aggressiveness, it lendscredence to the idea that violent games may in fact bethe root of violent behavior (as opposed to games ingeneral—get the idea?). A strengthener.

(D) The child psychologist’s first sentence suggeststhat (D)’s “older children” do fall within the argument’sscope, and if the playing of violent games is the tippingpoint between their less and greater likelihood offinding violence acceptable, then perhaps the link isvalid after all. A strengthener.

(E) actually reinforces that subtle assumption beingmade in the argument: namely, that aggressivebehavior implies the belief that aggressiveness isacceptable. When a choice bolsters an argument’sassumption, then by definition it’s a strengthener.

5. (A) Logical Flaw

Attacking the person who makes an argument has noimpact on the strength of that argument.

The letter-writer implies that the reason that ecologicalcriticism of wealthy nations’ lifestyles “should not betaken too seriously” is because its purveyors arehypocrites: they’re celebrity bigshots whose ownlifestyle is profoundly ecologically destructive.However, attacking a speaker as a hypocrite isexposing a moral flaw, not a logical one. The moviestars’ criticism of middle-class lifestyles may havevalidity over and above the stars’ personal behavior. Informal terms, the letter-writer has engaged in an adhominem argument, and though you don’t need toknow the Latin term you ought to recognize the problemas laid out by (A). Impugning the source of aconclusion has no effect on the conclusion itself.

(B) is outside the scope because the letter-writer’scriticism of bigmouth celebrities isn’t specificallydirected at their “sincerity,” but rather at thedisjunction between the lifestyle they follow and thatwhich they believe others should follow. The celebritiesmay be quite sincere in their beliefs, even if their ownbehavior is a major blind spot.

(C) The letter-writer does believe that the celebrities’viewpoint is “unreasonable,” but that’s because oftheir hypocrisy, not because they offer inadequateevidence, as (C) would have it. In fact, the celebrities’argument—that middle-class lifestyles causeecological damage, and therefore are open tocriticism—may indeed be adequately set forth.

(D) There is no evidence present that’s meant tosupport a point but actually weakens it, so the letter-writer can’t be faulted for “failing to recognize” same.

(E) The letter-writer pinpoints the hypocrisy of aspecific class of people—movie stars and othercelebrities—but makes no attempt to generalize abouteveryone.

6. (C) Weaken the Argument

“Overlooked alternatives” are an excellent way toweaken many arguments.

A particular explanation is not “the only one available”

Section II: Logical Reasoning

15

Page 18: LSAT_PT 49_Expl_web

if a plausible alternative can be identified, and that’swhat (C) does. It’s equally possible—if not more so—that the egret hangs around cattle for protectionagainst predators, as for the opportunity to eat stirred-up insects.

(A) That other birds may eat the bugs stirred up by thecattle doesn’t mean that that isn’t the sole motivationfor the egrets’ habit. Perhaps there are enough bugsfor all.

(B) offers other types of animal that cattle egretsfollow, but that has nothing to do with the motivationfor following any type of animal, cattle or otherwise.

(D) The issue is not where cattle egrets live, but theexplanation for a particular cattle egret behavior. Theseare very different issues.

(E) is even worse than (D); no offense if you chose it.If anything, the idea that forests are inhospitable toegrets because they can’t consume insects therelends strength to the argument that cattle egrets followcattle so that they can eat more bugs.

7. (E) Assumption (Formal Logic)

In a Formal Logic stimulus, not every statementneeds be relevant to the conclusion.

With the negative second sentence translated into thepositive:

All inspected fruit is uninfected.

...and the conclusion being:

All inspected fruit is safe to eat.

...we can see that all we need is a statement thatconnects the two mismatched terms, namely safety anduninfectedness, and that is (E). If you insert (E)(rewritten as “All uninfected fruit is safe to eat”), thesyllogism works perfectly. Sentence one has no impact,partly because it introduces a term (rottenness) thatnever reappears, and mostly because its emphasis ison identifying bad fruit, when the conclusion seeks toestablish the terms sufficient for good fruit.

(A) asserts the unsafeness of fruit that is rotten, butsince the conclusion is trying to establish when fruit issafely edible, (A)’s statement gets us no further along.The term “rotten” appears in sentence one but isirrelevant to the conclusion as stated.

(B), like (A), has no impact because the concept ofrottenness is more than a step removed from theconclusion.

(C) presents a hypothetical (“If such and such hadhappened, then thus and so would have resulted”) towhich one can only respond, “Well, it wasn’t inspected,so it wasn’t safe; and anyway we’re interested in theinspected fruit. So hush up.”

(D) actually offers the negation of correct choice (E).We cannot derive (E)’s “If uninfected, then safe” from(D)’s “If infected, then unsafe.” One can never negatethe two terms of an if/then statement and assume thatthe result must be true. (That’s a logical errorcommonly tested on the LSAT.) In any case, (D) asworded leaves us musing: “Okay, so infected fruit isunsafe. But we want to conclude something about safe-to-eat fruit, so you, choice (D), are no help.”

8. (D) Weaken the Argument

Arguments by analogy are inherently weak, and arebest attacked with evidence that the analogy isinappropriate.

What’s good for the cola is good for the call, thiseditorial seems to think. Soft drinks and phone callscost the same in the ‘70s; now 20 years later (note thedate on the editorial) the cost of the former hasdoubled, so why shouldn’t that of the latter? Becausesoft drinks and phone calls are manifestly differentthings, that’s why. The analogy falls apart if onespeaks to that difference. There may be logicalreasons why soft drinks cost more than phone calls,and (D) provides a good one: the greater cost ofproduction. (D) renders the analogy moot.

(A) might be true, but the issue is a cost borne by aconsumer of a soft drink or a phone caller in 1990, nota cost borne by a equipment owner/lessee in 1970.

(B) reflects a general trend over two decades. The costof a phone call (if a call properly falls within thecategory of “goods”) was an exception to that trend,and perhaps there were good reasons for that. (B) hasno effect on the logic.

(C) To choose (C) as the weakener, you must do morethan reject correct choice (D). You have to makeseveral unwarranted assumptions: that increasedgovernment regulation must add to the cost of phoneservice; that those costs would be passed along to theconsumer; and that in turn, a flat period of governmentregulation by definition means flat consumer costs.Surely this all strikes you as far too much work torender a choice acceptable.

(E) If you accept that greater sophistication ofequipment must mean higher development andoperational costs—which we shouldn’t, no matter how

PrepTest 49 Explained

16

Page 19: LSAT_PT 49_Expl_web

reasonable it sounds, since there’s no evidence fordoing so—then (E) might explain why a rise in the costof a phone call might be justified, i.e., to offset thoseincreased costs. But even if all that is so, (E) still failsto speak to the soft drink/telephone analogy that is atthe editorial’s heart.

9. (B) Inference

LR Inference questions are usually based on a set ofpremises, not an argument. Yet often the right answeris essentially a summary, i.e., the “main idea” of thefacts presented.

If, as we’re told, larger species are more vulnerable toextinction than are smaller species on the grounds ofthe relative amounts of food each needs to survive,then species survival must have at least someconnection to the food source of each individualmember of that species (since, of course, eachindividual eats its own food, or starves, depending onprevailing conditions). (B) offers a summary of thisidea, “vulnerability...at least in part” acting as a niceparallel to the stimulus’s “This fact helps make...morevulnerable.”

(A) Nothing in the stimulus supports any contentionabout any particular behavior being “the main factor” inany particular phenomenon. Indeed, the tone of thestimulus (“helps make...more vulnerable”) is quitetentative.

(C) sets the extinction of a large species as anecessary condition for the extinction of a smaller one.Say what? There’s no hint of that kind of connection inthe stimulus, not to mention the fact that (C), like (A),is too certain in its tone. “No small-animal species willbecome extinct” is far too unqualified an assertion.

(D) is in a sense self-evidently true: for the humananimal as well as any other, of course our survivalhinges on how much food we require. But that truismdoesn’t follow from this set of statements, which areon the topic of species survival generally, not the abilityof any one creature to survive. (Also, (D)’s “dependprimarily on” is no more justified than (A)’s “the maindetermining factor.”)

(E) draws a distinction that’s way out of line. It shouldmake sense that during an environmental catastrophe,a large species is at greater risk than a small one,because the latter needs less food to survive. That’s afar cry from (E)’s confident prediction that if the bigcreature is threatened, the small one will pull through.If things are bad enough, maybe all species of all sizeswill be wiped out; who knows?

10. (E) Point at Issue

Speakers can only disagree on terms that arecommon to both, so you can reject choices that dragin new terms.

The Kaplan Decision Tree is, as always, our anchor onPoint at Issue questions, but apart from that it’s worthnoting that three of the choices—(B), (C), and (D)—allmention a term, rationality/irrationality, that neverappears in either Megan or Channen’s statements.Since neither speaker opines as to that which isrational or irrational thinking, none of those choicescan possibly reflect their point of disagreement!

(A) The Decision Tree helps us right away. Meganoffers a condition necessary (“only if”) to pursuingwealth beyond their basic needs. That does not commither to the belief that some people do pursue wealthbeyond their needs. So in fact, we cannot be surewhether Megan even believes what (A) is saying.Eliminate! And we have already eliminated the nextthree choices for the reasons described above.

(E) is left over and must be correct. In fact Meganexplicitly says “Indeed, the desire for prestige andstatus is the only reason people pursue wealth beyondtheir needs.” Meanwhile, Channen says, “Noteverybody thinks that way”; those who are “indifferentto what others think” (and who therefore care not forprestige or status) can amass more money than theyneed, essentially telling us that there are otherreasons for people to pursue wealth beyond theirneeds. A flat-out disagreement, as required by thequestion.

11. (C) Strengthen the Argument / EXCEPT

Strengthening an argument makes the conclusionmore likely to follow from the evidence; it doesn’thave to prove that conclusion.

Lipoproteins start out as the villains of the piece (theytransport the dreaded cholesterol), but it turns out thatwhile LDLs are bad, HDLs are good—not just good, butthe subject of the “tentative conclusion” that HDLshelp prevent stroke and heart disease. The evidence isthat two factors positively correlating with lower riskthemselves correlate, at the same time, with highHDLs. Four of the five choices will serve to make thatconclusion somewhat less “tentative.”

(A) The ability of HDL to help excrete a “known factor”in stroke and heart disease strengthens the conclusionexplicitly. Eliminate.

(B) looks at the exercise/HDL/disease connection

Section II: Logical Reasoning

17

Page 20: LSAT_PT 49_Expl_web

from another angle. If (as it says) low levels of HDLcorrelate with obesity (and poor health), whereas highlevels of HDL correlate with solid exercise (and goodhealth), then the connection between high levels ofHDL and health is reinforced. Eliminate.

(C) The ease of removal of HDLs vs. that of LDLs isoutside the scope here; none of the evidence about theheart-healthiness and stroke prevention of HDLs isconnected with whether lipoproteins are ever removedat all. To make (C) work, you have to make a lot ofextra assumptions, including that both types oflipoprotein are removed from the blood, and that thegreater ease of removal of HDLs means that theremust be more HDLs and fewer LDLs around to affecthealth. One has to work way too hard to confirm (C)’simpact on the conclusion, so we can tell that (C) is infact correct. For the record:

(D) asserts not a correlation, but a directly health-improving effect on the part of HDLs: namely, that theymitigate the detrimental effects of LDLs. This could bethe surest strengthener of them all, really.

(E) The argument states that women have both higherHDL levels and less risk of heart disease and strokethan men in general. By asserting that those menenjoying a woman’s level of HDLs also enjoy a reducedrisk of heart disease and stroke, (E) may not prove thebenefits of high HDLs, but it certainly makes themmuch more likely. So (D) and (E) do strengthen thelogic, as we would expect once we recognized (C) ascorrect.

12. (C) Role of a Statement

Every statement’s role is to serve the conclusion, orat least to relate to it, in one way or another. So startthere.

The argument ends (“Thus”) with a paradoxicalconclusion: Bankers’ efforts to hold back inflation inthe short term may make it harder to hold inflationback in the long term (without angering the public, thatis). The author gets there by first describing howbankers try to curb inflation (that’s the first clause, theone whose role we’re asked about), and thenexplaining the results of the lag time between raisinginterest rates and restraining inflation: if you apply thebrakes too soon, you can succeed, but the effort isseen as holding back the economy to no purpose.Anyway, the line of reasoning is kicked off in the veryfirst clause, so (C) is pure and simply correct: that firstclause is one of several premises—that is, pieces ofevidence—leading to the conclusion.

(A) Since all the first clause does is explain themechanism by which bankers try to halve inflation, itcan’t possibly be a “complete explanation” covering allthe terms of temporary versus long term success, aswell as the public’s reaction to same.

(B) If the conjunction linking the stimulus’s first twoclauses were “because,” then sentence 1 would beworking as (B) suggests: the first clause would indeedbe a phenomenon explained by the second. Theconjunction “but” tells us that the two clauses areactually offered in contrast or opposition to each other.

(D) uses different language from (B)(“conclusion/support” versus (B)’s“phenomenon/explanation”) to assert the very samething: that clause 1 is supported by clause 2. Not so.Again, the Contrast Keyword “but” separating theclauses would not permit that.

(E) The entire stimulus is proceeding to the “Thus”clause. Contrary to (E)’s implication, the stimulus’sthird sentence is not a subsidiary conclusion, as the“but” in line 6 proves.

13. (E) Logical Flaw

Whenever you encounter an assertion of cause-and-effect, remember the three ways in which a causalrelationship can be weakened.

We hope that you recognized the causal conclusion,i.e., “The secretaries’ positive attitude caused orbrought about the excellent job performance.” True,there’s a strong correlation between their attitudes (asgauged by their survey responses) and performance(as gauged by their supervisors’ ratings). However, ifthe causal relationship is actually reversed, as (E)asserts it could be, then the correlation need not beinterpreted in the causal way that the speaker insists.Thus (E) is where the flaw resides.

Remember: a causal connection that “X causes Y”might work in reverse (as this one does: Y here couldbe leading to X); or there may be an independent causeZ for one or both effects; or the relationship between Xand Y could be totally coincidental.

(A) Since this conclusion is only about the secretaries,it certainly is not an improper generalization to all otherkinds of job performance.

(B) As noted above, there certainly is evidence:correlative evidence. It’s not persuasive orunimpeachable, but it’s there.

(C) “Other activities” beyond the sheer performance ofsecretarial duties are outside the argument’s scope.

PrepTest 49 Explained

18

Page 21: LSAT_PT 49_Expl_web

(D) Actually, no: “positive attitudes” is used to meanthe same thing all three times it’s mentioned, that is,“agreement that one enjoys one’s standard or new-found skills.”

14. (B) Weaken the Argument

Always be sensitive to assertions of necessity orsufficiency. Therein is where you’ll often find anargument’s Achilles heel.

Why must our ancestors have stood upright beforedeveloping those sophisticated tools? Because, saysthe scientist, sophisticated toolmaking “requires freeuse of the hands”—that is, free hand use is anecessary condition for sophisticated toolmaking. Sowhere does standing upright shine in? Well, it enablesfree hand use; it’s sufficient for free hand use.

The problem, of course, is that we have no ideawhether standing upright is the only posture thatmakes free hand use possible. If our ancestors couldhave had free hand use when sitting or lying down, ornever even getting up on their feet, then the connectionbetween standing upright and sophisticated toolmakingwill have been severed. (B) doesn’t describe theposture of those “advanced hunting weapons” makers,but it doesn’t need to. Simply by affirming thatsophisticated tools were evidently made by ancestorswho didn’t stand upright, (B) has weakened the bondbetween the evidence and the scientist’s conclusion.

(A) The development of basic tools is outside thescope. It’s the conditions required for sophisticatedtools that are at issue.

(C) The author assumes (incorrectly, as we’ve seen)that standing upright is necessary for developingsophisticated tools, but he never claims that standingupright must, necessarily, lead to sophisticated tools.Thus the assertion that some ancestors stood up butdidn’t make tools, which is (C), has no effect on hisreasoning.

(D) The concept of dexterity, relative to others or not,plays no role in the argument and thus (D) cannot bethe weakener. One can only weaken an argument on itsown terms; dexterity is not part of the terms here.

(E)’s tools may not have required standing upright, butthe ancestors’ ability to stand upright (if any) need nothave gotten in the way of those tools’ development. Inother words, since the toolmakers may have stoodupright anyway, (E) is consistent with the logic ratherthan weakening it.

15. (C) Principle (Strengthen the Argument)

A principle often acts like an assumption phrased ingeneral terms—it must connect the evidence to thekey terms of the conclusion.

The greater importance of testing household productsfor safety rather than Rx medicines—which is theauthor’s conclusion—is based on the fact that theformer are used by more people than the latter, and thepremise that a potential health risk increases as moreand more people use a product. The concept of safetytesting appears only in the conclusion, not at all in theevidence, so we have to connect a greater need forsafety testing with a greater risk posed by a popularproduct category, and that’s exactly what (C) does.Since the ordinary household maintenance productsare used by more people, they pose the greater healthrisk and hence, according to (C) (and the author) aremore important to test.

(A) implies that the question is “to test or not to test”a single product, when in fact the argument posing thequestion, “Which is more important to test for safetylawn chemicals or Rx drugs?” (A) does not evoke thatcomparison and so cannot have any impact on it.

(B), like (A), does introduce the topic of safety testing,but only insofar as it lays out the conditions underwhich such testing is important—period. (B) addsnothing to the comparison that is at the heart of theconclusion under consideration.

(D) focuses on a distinction between Rx medicines(those that need frequent doses vs. those that don’t),but that isn’t the comparison with which the author isprincipally concerned—the comparison between suchmedicines and everyday household products.

(E) begins with an assertion that is at odds with theconclusion (i.e. that medicines are more important totest than everyday products), followed by an “unless”exception, which the stimulus meets: more people areindeed at risk from household products than from Rxmedicines. But we’re left wondering: what now? If the“unless” exception is met, what conclusion can bedrawn? (E) falls crucially short of relevance. (Anotherproblem with (E) is that it focuses too narrowly on acomparison of one medicine versus one product,which ignores the stimulus’s interest in comparing twoclasses of product.)

Section II: Logical Reasoning

19

Page 22: LSAT_PT 49_Expl_web

16. (D) Inference (Formal Logic)/“could be trueEXCEPT”

When working with formal logic, always start with themost concrete, definite statements; save the moreslippery ones for later.

Starting with the most concrete statements: Ifeveryone who works 18+ hours a day has no time forleisure activities (that’s the second clause, rewritten inpositive terms), and if, clause 3, all happyentrepreneurs have time for leisure activities, thenthere are two groups that cannot overlap: thoseworking 18+ hours a day, and happy entrepreneurs.That impossible overlap makes (D) correct for this“could be true EXCEPT” question. There cannot be anyof the type (D) describes, because all the happyentrepreneurs have time for leisure activities, while allof those working 18+ hours a day do not.

Clause 1, the “most” statement, offers the fact that amajority of successful (a new term!) entrepreneurswork 18+ hours a day. This isn’t very precise, becauseit leaves room for all sor ts of successfulentrepreneurs, though in the minority, to work manyfewer hours than 18, and have time for leisureactivities and be happy, to boot. What this first clausemost accomplishes is setting up the wrong choices.

(A) could be true, because nothing stands in the wayof a 1:1 identity between those working 18+ hours aday and those with no time for leisure activities. Thosetwo groups could be one and the same.

(B) actually has to be true. (B) is speaking of peoplewho, according to clause 1, are in the majority ofsuccessful entrepreneurs—namely those who work18+ hours a day.

(C) could be true: there could be entrepreneurs whoare both happy and successful. They would be part ofclause 1’s (implied) minority: those entrepreneurs whoare successful yet work less than 18 hours a day(which leaves them available for the leisure activitiesthat correlate with happiness).

(E), even more than (C), speaks of clause 1’sminority. While most successful entrepreneurs work18+ hours a day, some certainly could work less.

17. (A) Assumption

The author’s central assumption must link the keyterms in the conclusion to the key terms of theevidence.

If we step back from the stimulus, we can recognizethat the author’s purpose is to consider the question,

“Do nonhuman animals possess consciousness?” Andwhile she doesn’t have a definitive answer, she does,in her conclusion, stake out a definite position: shebelieves that you cannot just prove that nonhumananimals are intelligent, and automatically conclude thatthey also possess consciousness. Her evidence issentence 1 plus the necessary assumption that will bethe correct answer.

Since she believes that the mere establishment ofintelligence is not sufficient to demonstrateconsciousness, she must be making some kind ofassumption that links intelligence—a term thatappears only in the conclusion—to the unique term inthe evidence, “exhibiting complex, goal-orientedbehavior.” The only choice making that linkage is (A),so you would be right to gravitate toward it and happyto realize that it’s correct.

Here’s how the logic works. Human beings, accordingto sentence 1, can lack consciousness at the sametime that they demonstrate complex, goal-orientedbehavior. If, as (A) asserts, a necessary condition forthat kind of behavior is intelligence, then what humanbeings represent are intelligent beings that (at times)lack consciousness. See that? Thus the conclusiondoes follow: you cannot simply establish a nonhumananimal’s intelligence and immediately jump to theconclusion that it has consciousness, because ashuman beings demonstrate, one can possess theformer trait but not the latter.

(B) is the inverse of the conclusion, which gets usnowhere. We need to connect the mismatched terms“intelligence” and “complex, goal-oriented behavior.”

(C) says that a necessary condition of consciousbehavior is intelligence, but the question at hand is,“Do nonhuman animals have consciousness?,” not“Are they intelligent?” In essence the argument istrying to reason toward consciousness, while (C) isreasoning from it.

(D) directly contradicts the conclusion: if possessingintelligence entails possessing consciousness, thenestablishing the former in nonhuman animals wouldestablish the latter as a nonhuman trait. Certainly theauthor isn’t assuming this statement to be true,because it denies her conclusion.

(E) draws an irrelevant distinction between “complex”and “goal-oriented.” Moreover, we need to connect (ascorrect choice (A) does) the very terms that choice (E)is determined to drive a wedge between.

PrepTest 49 Explained

20

Page 23: LSAT_PT 49_Expl_web

18. (E) Logical Flaw

A complex, verbose stimulus needs to be reduced toits bare bones.

This peroration is awash with adverbs and adjectivesand repetition that can set your head spinning if youdon’t take pains to reduce the logic to its essence.Starting with the Conclusion Keyword “therefore,” youmight formulate something like this: The bestunderstanding of nature requires nontraditional(holistic) reasoning, rather than traditional (scientific)reasoning. That’s because nature evolves in a holisticway and is itself holistic and interconnected.

This kind of distillation may reveal the author’sunwarranted scope shift between the holisticcharacteristics of nature mentioned in the evidence,and the holistic reasoning for which the author arguesin the conclusion. And that should lead you to (E). It’san unwarranted assumption that to best understand aholistic phenomenon (like nature) one must applyholistic reasoning; for all we know, a traditionally linearreasoning approach might be equally adequate to thetask.

(A) accuses the argument of confusing necessity andsufficiency. While that’s a flaw plaguing many LSAT LRarguments, this isn’t one of them, as our distillationabove demonstrates. The problem is not thesufficiency of evidence but the inappropriateness ofevidence: that is, a conclusion about reasoning needsevidence about reasoning, not generalizations aboutthe essence of nature.

(B) The philosopher doesn’t argue that linearreasoning cannot come to grips with nature, simplythat it falls short of the best way to do so becausenature is organic rather than linear. (B) is on the righttrack, but since the author could concede a possibledifference between the structure of nature and thestructure of thinking, without compromising hisinsistence that holistic thinking is the best approach,(B) doesn’t point to the argument’s vulnerability.

(C) The argument cuts right past any distinctionbetween parts and wholes—almost ruthlessly so. Onecan only attack an argument on its own terms, and theterms of this argument are the leap from the holistictraits of nature to a holistic approach of analysis. Don’tbe fooled by (C)’s use of terms from the argument!Just because the author mentions something doesn’tmean it will be in the right answer.

(D), like (C), appeals to a specious and irrelevantdistinction between the whole and the parts.

19. (D) Assumption

Always look to the mismatched terms in order tolocate an assumption.

“Fairness” is the term in the conclusion that appearsnowhere in the evidence, so to reach this conclusionwe need, somehow, to connect the behavior of theowners (the owner is the “someone else willing to payfor” athletes’ salaries). (D) satisfies this requirementnicely. If (as (D) says) any salary that an owner iswilling to pay is fair, and if (as we’re told) the salariesathletes are currently earning are determined by willingowners, then yes, one would have to conclude that thesalaries are fair.

(A) opines on the fairest “economic system,” which isbroader than the narrow conclusion about fairness inpro sports. (A) could be true, but still leaves open thequestion as to whether any particular salary—such asthe athlete’s—is in essence “fair.”

(B) focuses on the most tangential element of theargument—indeed, it’s part of the paragraph, but notreally part of the argument at all—and that is themotivation (i.e., “enormous profits”) behind theowners’ largesse. The fairness issue is totallyseparate; any loss of profits that might follow loweredsalaries has nothing at all to do with fairness.

(C), like (B), fails to address fairness, which is a biggiveaway. And (C), even more than (B), focuses on theutterly irrelevant issue of why the owners are ponyingup so much cash.

(E) reasons from the fairness of athletes’ salaries inits “if” hypothetical, and that offers the argument—which is trying to establish that fairness in itsconclusion—no help whatsoever.

20. (B) Inference

Often the line between an Inference and an author’sconclusion is fine indeed.

The environmentalist paints a grim portrait of whathappens when trash is incinerated: poison from heavymetals escapes into the air. And since manyappliances contain heavy metals, surely incineratingthem would be a mistake: that’s (B), a restatement ofthe danger cited in the environmentalist’s firstsentence.

(A) You may know that recycling is a real-worldalternative to incineration, but since the term is nevermentioned, (A) can’t possibly qualify as a statementthat “must be true” or “must follow.”

Section II: Logical Reasoning

21

Page 24: LSAT_PT 49_Expl_web

(C) Clearly the author thinks that chlorofluourocarbonsare dangerous, because she cites their presence inrefrigerators right after saying “Discarding oldappliances can be dangerous:”—and note the colonthere. Trouble is, chlorofluorocarbons aren’tnecessarily harmful to the atmosphere, as (C) wouldhave it. Maybe they are just an earthbound nuisance.Insofar as this stimulus is concerned, we only know ofheavy metals’ threat to the atmosphere.

(D) For all we can tell, newer appliances may containequal amounts of heavy metals as older ones, andhence pose exactly the same threat. (D) is a classicirrelevant comparison.

(E) is a 180. On the evidence of the passage,landfills—at least well-operated ones—are a far betterfinal resting place for appliances than the incinerator.

21. (B) Inference / “could be true EXCEPT”

If an answer choice is outside the scope of astimulus, then it certainly “could be true.”

The purpose of this stimulus overall is to assess theeffects of sugared beverages. On the plus side, they’retasty so they help to forestall dehydration, and smallamounts of sugar have the positive effects ofenhancing water absorption, delaying muscle fatigue,and maintaining glucose level. But on the minus side,too much sugar can make dehydration worse. We’relooking for a choice that contradicts the above.

(A) Glucose is the only sugar mentioned here, butthere’s no impediment to there being other sugars thatcan fatigue the muscles if they—the sugars—happento be scarce. This statement is possible. Eliminate.

(B) Dehydration problems are “invariably” exacerbatedif a muscle-fatigue-delaying substance is consumed?No: we have heard that sugar, which can delay musclefatigue, “can be helpful in avoiding dehydration”; it’sonly when consumed to excess that the reverseoccurs. This is the statement at odds with thestimulus; (B) is correct. For the record:

(C) How sweet is “too sweet”? The author never dealswith this question, so (C) is certainly possible. Indeed,since large amounts of sugar make one moredehydrated, (C)’s reaction could well be the body’s wayof saying “Don’t drink this now; you’re dehydrated.”

(D) This stimulus is about a substance that can delaymuscle fatigue. Since substances and situations thatincrease muscle fatigue are outside the scope, then(D) can certainly be true.

(E), too, concerns an outside-the-scope issue. We

don’t know enough about the interplay of watercontained and absorption rate to contradict (E), so it isone more statement that “could be true.”

22. (E) Assumption

Take the time to reduce a complex argument tosimpler terms.

This conclusion (“Hence,”) is an example of arecommendation against a policy: namely, the policy ofaccepting computer-assisted math proofs that offer avast “number of instances.” The author’s problemseems to be the dichotomy between a necessarycondition for acceptance (namely the need to review,independently, every calculation in a proof), and thehuge number of calculations that a computer canperform, which are many more than any human couldreview independently. Set up in this way in your mind,you may more likely ask a key question: “Why couldn’ta computer per form a satisfactory independentreview?” The author must believe that there’s a reasonwhy not, otherwise he’d leave open the possibility of a“reviewing machine” applied to a “proof-makingmachine.” The assumption that another computercouldn’t perform the adequate review is laid out in (E).

(A) is a definite advantage in the use of computers, butdoesn’t speak to any of the conditions surrounding theappropriateness of accepting a computer-assistedproof. That very “simplification” could be a blow to theproof’s correctness.

(B) That most proofs don’t attempt to demonstratetruth is irrelevant. In this argument, the demonstrationof veracity is the purview of the independent reviewer,not the “proof-maker.”

(C), like (A) and (B) before it, concerns only thecreation of a proof, not the independent review of one.And again, the perceived impossibility of independentreview is what leads to the author’s recommendation.

(D) The scope here is computer-assisted proofs only.The moment we read (D)’s first 10 words we areentitled to reject it.

23. (B) Logical Flaw

When an author “presumes the conclusion to betrue,” he is offering no independent evidence for it.

To prove that human behavior requires consideration ofpeople’s nonphysical elements, the commentatorargues that even if every physical element of an actionwas catalogued and understood, we still wouldn’t fullycomprehend the action. On what evidence? The

PrepTest 49 Explained

22

Page 25: LSAT_PT 49_Expl_web

commentator is essentially arguing that “we can’t fullyunderstand people on a totally physical level, becausewe can’t fully understand people on a solely physicallevel.” He provides no independent evidence for thatlack of understanding (all he says is “obviously” we stillwouldn’t understand), and that’s what makes theargument feel like it’s going in a circle.

Indeed, this is a classic example of “circularreasoning,” in which the author presumes hisconclusion to be true in order to prove it. You’re notresponsible for the term, but you should spot the flawwhen it appears. (B) describes it in appropriate,abstract terms.

(A) There’s no analogy here. An analogy is acomparison of two essentially different things. Herethe body is analyzed from the perspective of its twocomponents: physical and nonphysical. No analogy.

(C) No, the circularity here is that it concludes that aproposition (the commentator’s first sentence) is truebecause it’s true, not because it’s been tested and noone has disproved it.

(D) A speaker need not be so open-minded as to be“aware of any evidence that could undermine” hisargument. He would be wise to consider it and defendagainst it in advance, but he’s not required to do so,so (D) is not describing the commentator’s logical flaw.

(E) The commentator asks us to “suppose” a completescientific cataloguing of a human action. This is notunlike a Logic Games question’s “if” clause, in whichwe are to suppose that G goes to the picnic, at leastfor that question only. In both cases, we are not toargue with the supposition, as (E) would have us do;we’re supposed to accept it as true and proceed fromthere. So the commentator is not “presuming” that thesupposition is true; he is asking us to presume that it’strue for the moment, in order to press his case.

24. (A) Parallel Reasoning

Characterizing an author’s conclusion can be useful inunderstanding a complex argument.

The topic here is the effect on deterring crime of a harshsentence, and the conclusion (“Thus,”) is asserting aparadox. There are two factors in deterrence: the severityof the penalty, and the likelihood that the convictedperson will have to suffer it. Problem: if the penalty is toosevere, the likelihood of suffering may disappear (jurieswill just acquit), and the very deterrence hoped for isdiminished. The right answer must offer determiningfactors, one of which, if mishandled or applied in excess,will undo the effect of the other.

(A)’s elements match up with those of the stimuluswith almost geometric precision. Success in getting thefirst job, like the deterring power of punishment, comeswith two determining factors. And in both cases, if onefactor goes too far (the severity of the punishment /the time spent working on the dissertation), thedesired outcome may be foiled. (A) is the correctanswer.

(B)’s evidence and conclusion both offer “likely”comparisons between situations, and they parallelnothing in the stimulus (which uses “likelihood” onlyonce, and in a very different context).

(C) Instead of two determining factors, (C) offers onepositive and one negative effect of the new surgicaltechnique. Not parallel.

(D) presents a conclusion in the form of arecommendation (“governments…should put theirenergies”), so different from the stimulus’sparadoxical cause and effect that you can reject (D) onthat basis alone.

(E) There’s no paradox in (E), and no pair ofdetermining factors that, if one appears in excess, canundo a hoped-for result. There’s just the easily-accepted premise that an unknown artist can chargemore as his reputation goes up. The conclusion (thatone can reverse the process) doesn’t really follow and(E)’s author commits a logical fallacy—all the morereason to reject (E), since the stimulus is guilty of nological error.

25. (C) Assumption

The need for an assumption signals that something ismissing—that the argument is not quite complete.

Cecile clearly does not meet either of her association’srequirements for public disclosure of investments: shecan’t disburse funds and doesn’t sit on the board of apetrochemical company. What would seal the deal isknowledge that there are no requirements other thanthose of her association that might demand disclosure.(C) makes that assumption explicit, and if there are noother reasons for her to disclose, then the conclusionis justified.

(A) The conclusion concerns Cecile’s need to disclose“at this time,” which means that the prospect, raisedby (A), that she might get disbursement authorizationpower later on is outside the scope.

(B) “Conflicts of interest” are never listed as animpediment to keeping one’s investments private.Outside the scope.

Section II: Logical Reasoning

23

Page 26: LSAT_PT 49_Expl_web

(D) makes the argument no stronger, if assumed,because Cecile is clearly not on the petrochemicalcompany’s board, notwithstanding that that companyowns the timber company on whose board she doessit. (D) doesn’t have any effect on the rules.

(E), like (B), mentions an issue that might be relevantto the disclosure of investments in the “real world,” butthat never comes up in the course of this argument.

26. (C) Parallel Reasoning Parallel Flaw

Defining the flaw in the stimulus can help you locatethe parallel answer choice more quickly andaccurately.

What is the evidence for the claim that those with high-fat diets “do not consume too many calories”? Simplythat obesity is caused by some other factor (it doesn’teven matter, for our purposes, what that factor is). Theproblem, of course, is that health and fitness issuesother than obesity may have a high-caloric cause; inother words, the subject could still be consuming “toomany calories.” A search for a syndrome or conditionthat the author denies while failing to realize that it maystill pertain leads us to (C). The fact that there’s analternative cause for post-flight disorientation doesn’tmean that the pilots don’t suffer from sleepdeprivation.

(A)’s conclusion parallels the stimulus’ claim that‘obesity is caused not by calories but by a nutrientdeficiency.’ Trouble is, the stimulus uses thatstatement as evidence, not as the main conclusionitself.

(B) should be instantly suspect because of itsconclusion’s use of “most” and attribution of a trait toa group of people. Both of these fail to parallel theconclusion that all members of a class (high-fat eaters)lack a trait.

(D) falls far short of denying a trait to a group; instead,it concludes with an assertion of fact about predictingeconomic downturns. (D) would be more parallel if itsconclusion were, “Therefore, there is no panic when aneconomic downturn is predicted.”

(E)’s terms are much more complicated than that ofthe original and choice (C), which basically say, “X iscaused not by Y, but by Z. Therefore, Y doesn’t exist.”By contrast, (E) traffics in statements about reasonsfor certain behaviors, and ends with a conclusion,“Therefore, most X have Y.” Very, very different.

PrepTest 49 Explained

24

Page 27: LSAT_PT 49_Expl_web

SECTION IIIREADING COMPREHENSION

Passage 1: Computer-Generated Displays

At long last, the Topic of this passage is somethingwe’ll almost certainly see in our legal careers: the useof computer-generated visual displays in thecourtroom. While that information appears in the firstsentence, the author takes much longer to reveal theScope. The entire first paragraph deals with whatexactly these computer-generated displays are, andhow their use in the courtroom provides certainadvantages, which might have led you to believe thatthe author would have a uniformly positive view of thisnew tool.

But paragraph 2 takes a different tack, introducingcritics who highlight potential problems with thedisplays, including manipulation of the jurors’ opinionsand an exacerbation of any financial disparitiesbetween legal teams. So our Scope has become morecomplex, and now includes both the benefits and theproblems of computer-generated visual displays.Paragraph 3 adds another element, the author’ssuggestions on how to avoid the pitfalls mentioned inparagraph 2. These suggestions give us an idea of theauthor’s Purpose: to evaluate the use of computer-generated visual displays and suggest steps tomitigate the problems associated with them. Eventhough the author clearly believes that the displays area useful tool, she argues in favor of taking steps toensure that they are not misused. In fact, that lastsentence would be a pretty good paraphrase of theauthor’s Main Idea, and it comes directly from the firstsentence of paragraph 3. “To avoid misuse of thistechnology in the courtroom, practical steps must betaken.” That “must” shows us where the authorstands.

Roadmap:

¶1—Computer-gen. visual displays: what theyare, why they’re good¶2—Critics: potential problems with PCdisplays¶3—Author: must take steps to avoidproblems. List of steps.

The Questions:

1. (A) Global (Main Idea)

Use the author’s statements of opinion to guide youto her Main Idea.

We want to tie together the two aspects of the author’sopinion on computer-generated visual displays: heracknowledgment of their “advantages” (line 15) andher advocacy of steps to avoid their misuse inparagraph 3. But we don’t want to focus on thepotential problems with the displays, since the authormakes it clear that the problems are especiallyimportant to “some critics” (26). Only (A) mentions theadvantages of computer-generated displays whilefocusing on the importance of taking steps to preventthe misuse of the displays, and it’s a close paraphraseof the first sentence of paragraph 3, which led us to theMain Idea in the first place.

(B) The author does tell us that the use of computer-generated displays “is growing” (line 2), but there’s nohint that it has “grown dramatically,” as (B) would haveit. Besides that, the author certainly doesn’t focus onthe reasons why the use of these displays has grown.

(C) is a classic half right, half wrong. It starts off well,giving us the author’s reason why computer-generateddisplays need to be restricted, but then veers off totalk about “the most sophisticated principles ofjurisprudence,” which are not a part of the passage.

(D) is a Faulty Use of Detail. It only looks at thepositive elements of paragraph 1, instead of continuingwith the author’s suggestion from paragraph 3.

(E) While this choice may have been tempting,it’sreally too extreme to be any good. The author neverclaims that the disadvantages can definitely beeliminated, only that certain steps are necessary toprevent them.

2. (D) Global (Organization of the Passage)

Your Roadmap is an excellent paraphrase of thecorrect answer to an Organization of the Passagequestion.

We’ve already got a great pre-phrase of the answer,because we were smart enough to follow the KaplanMethod for Reading Comprehension. The passage startsby introducing computer-generated visual displays andtalking about their advantages, then the critics come inand talk about potential problems, then the authorsuggests steps that will help avoid those problems. (D)matches our Roadmap almost word-for-word.

Section III: Reading Comprehension

25

Page 28: LSAT_PT 49_Expl_web

(A) The author never “laments” the popularity ofcomputer-generated displays.

(B) We could probably eliminate this choiceimmediately, since the author doesn’t really endorsethe new technology in the first paragraph. But if herinitially positive attitude led you to continue checkingthrough (B), the omission of the negative aspects ofthe new technology should have shown you that thechoice was incorrect.

(C) skips right over the first paragraph. The problemsof the technology aren’t discussed until paragraph 2.

(E) Saying that “a new technology is described indetail” doesn’t really capture the positive slant ofparagraph 1, but again, it’s not so clearly wrong as toallow you to eliminate (E) quickly. This choice goeswrong when it says that the author’s finalrecommendations are aimed at promoting the use ofthe new technology instead of preventing its misuse.

3. (E) Inference

Don’t let an atypical question stem confuse you;you’ll be fine as long as you can focus on what thequestion asks for.

This question doesn’t look like a typical Inferencequestion, since it doesn’t ask us for something that theauthor or a critic would be most likely to agree with, orsomething suggested by the passage. But it does havethe loose language typical of Inference question stems,asking for an answer choice that is “most similar” to re-creating an accident with a computer-generated visualdisplay. That means we can treat this oddball questionjust like a regular Inference question: we’ll go back tothe passage and do our research, paraphrase an answerthat must be true based on the information we have,then hunt through the answer choices for a match.

Our Roadmap points us back to paragraph 1: that’swhere we find the courtroom use of computer-generatedvisual displays discussed in detail, before the authordiscusses the disadvantages and her recommendationsfor the use of such displays. The question stem’smention of “re-creating an accident” further refines oursearch to lines 13–15, which suggest that visualdisplays “creat[e] the illusion that viewers are at thescene of a crime or accident, directly experiencing itsoccurrence.” We’ll search for an answer choice thatwould do something similar:

(A) suggesting a motive wouldn’t be at all like re-creating the scene of the crime. Eliminate.

(B) Again, there’s no illusion created here—only anediting process. Eliminate.

(C) This choice’s “satellite images,” which are likelysimilar to computer-generated displays, may have ledyou to select it. However, they are used to predict thefuture, not re-create the past. Eliminate.

(D) Again, the type of evidence might have fooled you—a video camera sounds a lot like a visual display. Butthe opinions of passersby wouldn’t create the illusionthat observers were directly experiencing a recentoccurrence. Eliminate.

(E) is the only choice left, so it must be correct. Sureenough, (E) uses evidence to re-create a past event—in this case, a volcanic eruption in a museum exhibit.This choice is correct.

4. (B) Inference

Some questions will require you to research the textusing Hot Words from within the answer choices, butauthor’s voice can help to guide us in broad Inferencequestions.

This question stem gives us a clue that doesn’t reallyhelp us find the correct answer. We have to find astatement that the author would agree with regardingthe use of computer-generated displays in thecourtroom, but practically the entire passage consistsof such statements. We’ll have to walk through theanswer choices one by one, using the information ineach answer choice to guide our research. We did,however, see the strongest statements of author’svoice in paragraph 3, so that might help us. To savetime with this approach on Test Day, remember thatyou should move on once you’ve found the correctanswer.

(A) The “financial aid program” for computer-generateddisplays appears at the end of paragraph 3. While theauthor definitely thinks that such a program “wouldhelp create a more equitable legal arena,” (60) shenever suggests that the courts should suspend the useof computer-generated displays until financial aid is inplace. In fact, financial aid was introduced as a remedyif one side’s resources were greater than the other; ifboth can afford such displays, there would be no needfor such aid. Fur thermore, “stop-action andhighlighting” are misleading techniques mentioned inline 40, so financial aid would be irrelevant to theirvalidity. Eliminate.

(B) Scrutinizing computer-generated evidence to checkfor excessive speculation sounds like a problemmentioned in paragraph 2 or a recommendation fromparagraph 3. In fact, it’s a combination of both. Lines43–45 suggest that speculation can make computer-

PrepTest 49 Explained

26

Page 29: LSAT_PT 49_Expl_web

generated evidence “unsuitable for use in a trial,” andthe author’s first suggestion in paragraph 3 is thatcomputer displays should be subject to “diligentanalyses” to avoid the misuse suggested in theprevious paragraph. The author would definitely agreethat (B) is true, so this is our correct answer. For therecord:

(C) and (D) both make irrelevant comparisons. “Actualstatic photographs of a crime scene” and “verbalaccounts by eyewitnesses” are outside the scope ofthe argument, as is the effectiveness or the role of anyevidence other than computer displays. Eliminate.

(E) The author never discusses unrealistic orunpersuasive displays—she is far more concerned withmanipulative techniques that could make misleadingdisplays more persuasive. Eliminate.

5. (C) Detail

Concrete language in the question stem means thatwe’re dealing with a Detail question.

Instead of something suggested or implied by theauthor, now we’re looking for something that sheexplicitly states in the passage. Once again, the clue inthe question stem is so broad that we’ll have to usethe Hot Words in the answer choices to assist us in ourresearch. Our Roadmap will still help us by directingour research to one paragraph or another.

(A) Any “advantages over conventional forms ofevidence” offered by computer displays would be foundin paragraph 1—a quick glance at the latter half of thisparagraph will find several, making (A) a 180.Eliminate.

(B) The critics of computer-generated evidence appearin paragraph 2. While their statements are describedas “urging caution in the use of these displays” (27),an argument in favor of an outright ban is nowhere tobe found. Eliminate.

(C) Anything that judges should do with respect tocomputer displays would fall into paragraph 3 with therest of the author’s recommendations for the use ofcomputer displays. A skim of this paragraph will turn uplines 54–56—(C) repeats these words verbatim. Thisis our correct answer. For the record:

(D) The current use of computer displays is describedin paragraph 1. Lines 21–22 say that the displays are“especially valuable ” in technical trials, but thisdoesn’t suggest that such trials are primarily where thedisplays are used. Eliminate.

(E) distorts the last few lines of the passage. The

author argues that “steps should be taken to ensure”equal access to computer technology (56–57), but notthat the litigators already using the technology shouldbe the ones taking those steps. Eliminate.

6. (E) Detail / EXCEPT

In an EXCEPT question, characterizing the answerchoices and eliminate your way to the correct answer.

In contrast to the previous question, the correctanswer to this Detail / EXCEPT question will be the oneanswer choice that the author did NOT mention in thepassage. The four wrong answer choices will all bethings mentioned by the author. We get one additionalclue in the question stem: the four wrong answerchoices will all be advantages of using computerdisplays in the courtroom. This allows us to direct ourresearch to paragraph 1, where the advantages ofusing this technology are listed. The only advantagementioned in an answer choice but not in paragraph 1is in (E); in fact, the second paragraph suggests thatone of the potential misuses of computer displays isallowing speculation.

(A) Lines 7–8 state that computer displays make itpossible “to slow or stop action.”

(B) Lines 19–22 discuss the greater understandingfacilitated by computer displays and its value in“complex or technical trials.”

(C) is mentioned as an advantage in lines 8–9.

(D) The final lines of paragraph 1 are repeated nearlyword-for-word in (D).

Section III: Reading Comprehension

27

Page 30: LSAT_PT 49_Expl_web

Passage 2: Centers of Style

This passage gets right to the point. According toparagraph 1, the techniques used for judging the tribalorigins of African art (Topic) have gotten better over thelast few years, but a critic has questioned their centralassumption, the idea that any particular style can betraced to one tribe. That criticism, and the author’sevaluation of it, will be the Scope of the passage.Paragraph 2 begins on to evaluate that critic’sassertion. This is where we first hear of the mostimportant concept in the passage: the “centers ofstyle” that produce art for distribution over a widegeographical area. The centers of style will form theentirety of the author’s evidence in favor of the critic’sassertion, and the Conclusion Keyword “thus” in line19 leads us to the author’s idea about these centersof style. According to the author (who seems to agreewith the critic from the previous paragraph), centers ofstyle would allow artists from a single ethnic group tocreate works of art for several neighboring tribes,confounding historians’ efforts to assign artwork toany one tribe.

From the general idea of centers of style presented inparagraph 2, the passage progresses in paragraph 3 toa specific example of one center of style, the Konatéfamily of Burkina Faso. This example provides evidencefor the difficulty of tracing artworks to a single tribe:the Konaté provide sculptures to five differentneighboring ethnic groups and to tourists (31–34) Eventhough these sculptures have subtle differences foreach tribe, they are largely consistent—thus, they area per fect example of ar twork that would defyhistorians’ efforts to link them to one or another of theclient tribes. It sounds like the author’s Purpose is toargue that historians of African art should consider thepossibility that centers of style were actuallyresponsible for certain similar artworks instead ofattempting to classify these works to specific tribes.

Paragraph 4 ties everything together, with a string ofsuggestions for historians of African art. Thesesuggestions together comprise the author’s Main Idea,that historians of African art should pay greaterattention to centers of style and consider thepossibility that certain African artists created art forwide distribution instead of solely for their own tribe.

Roadmap:

¶1—Critic: classification techniques for Africanart maybe based on false assumption¶2—”centers of style” distribute art, confusetribal classification¶3—Example: Konaté family = center of style¶4—M.I.: Historians of African art shouldconsider centers of style.

The Questions:

7. (A) Global (Title)

A title for the passage must include the entire scopeof the passage, just like the Main Idea.

We can form a strong pre-phrase of the answer to thisquestion by simply considering the scope of thepassage. The correct title must consider centers ofstyle, and it must mention their implications for theclassification of African art. As it turns out, only (A)mentions even the first of these crucial elements, so itmust be the correct answer.

(B) Line 34 mentions “the tourist trade” in passing,but this brief note hardly merits mention in the title ofthe passage.

(C) “Proportion, composition, color, and technique”are the consistent characteristics of Konaté sculpturefrom line 42, making this choice too focused on thedetails.

(D) is a bit too extreme: the passage really isn’tagainst historians of African art, it merely suggeststhat they’ve failed to consider an important fact aboutAfrican art.

(E) While the Konaté are the only ar t tradersmentioned in the passage, they are merely used as anexample of a center of style, and are not really thefocus of the passage itself.

8. (E) Inference

An Inference on the LSAT is always something thatmust be true.

Even though the art historians don’t get to presenttheir side of things, they are an important part of thepassage. This question asks us to infer their viewpoint,or at least a viewpoint they would agree with, and theline reference in the question stem suggests that we’llfind it at the beginning of the passage. Skimming the

PrepTest 49 Explained

28

Page 31: LSAT_PT 49_Expl_web

first few lines for context, we can find a reference tothese historians refining the techniques that they usein “judging the precise tribal origins of Africansculptures (2–3).” If they’ve spent the last half centuryworking to refine those techniques, then surely thehistorians believe that there is some value in theinformation gained from them—so (E), which stressesthe importance of that information, must be true, andis correct.

(A) is a classic example of a 180. It gives us theauthor’s viewpoint, not the art historians’.

(B) The techniques for carving the eyes and mouths ofmasks are mentioned in lines 37–39 as features usedby the Konaté to distinguish tribal styles. Thesetechniques are not “standard”: they are crafted to suiteach tribe’s specifications. Furthermore, we don’t haveany indication of the art historians’ views regardingtechniques.

(C) is another 180; the author argues that substylesshould not be distinguished from the regional stylescreated by centers of style, but the art historians aredescribed as “attempting to break down large regionalstyles into finer and finer tribal styles and substyles(51–53).”

(D) The only example of mask sculptors producingmasks for other tribes comes from the Konaté, but thepassage states that they do so as part of “a longtradition” (31). It might be possible that the arthistorians agree with (D), but in doing so they wouldcontradict the passage, so we can’t say that (D) mustbe true.

9. (E) Detail

You should be able to form a very specific pre-phraseof the answer to most Detail questions.

The LSAT has given us a gift in the form of the stem forquestion 9. We won’t every get a clue that’s muchmore specific than “a feature that Konaté sculptorscan identify as a requirement of a particular tribalstyle.” This hint leads us to lines 37–39, which alludeto “the foliate patterns that radiate from the eyes of aNuna mask, or the diamond-shaped mouth of many Komasks.” The second distinction, mouth shape, is (E).

(A), (C), and (D) None of the characteristicsmentioned in these three choices are mentionedanywhere in the passage.

(B) might have thrown you for a loop, given thedistinguishing characteristics of a Nuna mask; but notethat Nuna masks are distinguished by patterns radiatingfrom the eyes, not the position of the eyes themselves.

10. (E) Global (Purpose)

The strength of the author’s point of view will bereflected in the strength of the verb used to describethe purpose of the passage.

The author of this passage has a particularly strongpoint of view—the art historians’ assumption is wrong,and they should rethink it. We’ll look through theanswer choices for verbs that might match the strengthof this point of view; only “argue,” in (E), even comesclose. A quick skim of the remainder of the choiceshows that it matches perfectly with the scope of thepassage—the author does indeed craft the passagearound the idea that the “particular approach toclassifying” African art used by the art historians isincorrect.

(A) There is neither a classification nor a “newlyproposed set of principles” in the passage.

(B) might be correct for the next passage that thisauthor writes, but it goes a step further than thepassage at hand. This passage doesn’t argue that anyparticular group of artwork has been misclassified,only that the classification methods of the past andpresent do not take into account an important aspectof African art.

(C) The passage does not explain the principles usedby the art historians to classify work—instead, itfocuses on an assumption underlying those principles.

(D) is tempting, but too weak. The author doesn’t justreveal the underlying assumptions of the traditionalapproach, he provides evidence that they are notalways correct.

11. (D) Inference

Questions that do not provide any clues in thequestion stem take a particularly long time to answer,so they are good candidates to skip.

We’ve got a classic Inference question here, so weknow that the correct answer is something that mustbe true based on the passage. Unfortunately, thequestion stem doesn’t give us any hints as to where inthe passage we’ll find the correct answer. We’ll have toevaluate each choice as we go, which can be very time-consuming. Fortunately, a quick skim of the answerchoices reveals that they all deal with the Konatéfamily of sculptors, so we can narrow our research toparagraph 3. But this will not always be the case—many questions that lack clues in the question stemwill still require you to research the entire passage. Ifyou have trouble finishing the Reading Comprehension

Section III: Reading Comprehension

29

Page 32: LSAT_PT 49_Expl_web

section on Test Day, this would be a good place toimprove your pacing by moving on to a question thatwon’t take quite so long. For now, we’ll evaluate eachchoice until we find the correct one:

(A) Sculptors outside of Burkina Faso are outside thescope of the passage, which only includes one specificexample of African artists. Eliminate.

(B) The passage makes distinctions between thesculptures made by the Konaté for other tribes, but notbetween the sculptures made by different members ofthe Konaté family. Eliminate.

(C) Other sculptors within Burkina Faso are alsooutside the scope of the passage. The Konaté are theonly African artists specifically mentioned by theauthor. Eliminate.

(D) Paragraph 3 tells us that the Konaté producesculptures “for five major neighboring ethnic groups(32–33),” so (D) must be true, and is correct. For therecord:

(E) Even though the passage tells us that Ouri has “along tradition of sculpture production (31–32),”meaning that it is certainly an old center of style, it isnot necessarily the oldest center of style in the entirecountry of Burkina Faso. Eliminate.

12. (B) Detail

Answer easier Detail questions before you answermore difficult Inference and Logic questions.

Even though this question was second-to-last in thequestion set for this passage, you should have done itmuch earlier than that. As a Detail question, it isalmost certainly easier than question 8 or question 11,both of which are Inferences. By answering the easiestquestions first, you can be certain to pick up points asquickly as possible.

The Konaté sculptors are found in paragraph 3, so wecan direct our research there. Four of the answerchoices will not be found within this paragraph or withinthe passage, while one of them will be a detail aboutthe Konaté sculptors taken from paragraph 3. As itturns out, (B) contains that detail—the Konaté “areable to distinguish the characteristics of the five stylesin which they carve (35–36),” but “the characteristicpatters are so subtly different that few people outsideof the area (44–45)” can do so.

(A) “Nontraditional materials” are not mentioned in thepassage.

(C) and (D) The only artists mentioned in the passageare the Konaté. Neither “other sculptors” nor a carving

style “used only by members of a different tribe” are apart of paragraph 3.

(E) While paragraph 3 does suggest that the Konatéhave been producing sculptures for a long time,referring to “a long tradition of sculpture production(31–32),” there is no hint that they introduced thepractice.

13. (C) Inference

Inference questions with direct text references oftenrequire us to combine and paraphrase two or morerelevant pieces of the text.

“Centers of style” is introduced in line 16, but it isused throughout the remainder of the passage, so theanswer to this question will most likely take severaldifferent references into account. Some research intothe context turns up a few clues. Centers of style aredefined early on as groups that “produce sculpture andother art that is dispersed over a large, multitribalgeographical area (17–19),” but this isn’t all theinformation we have about centers of style. The Konatéare once again central to finding the correct answer, asan in-depth example of a center of style. A descriptionof the Konaté will provide us with another element tosearch for: while their sculptures have five distincttribal styles, it is nevertheless “consistent in itsproportions, composition, color, and technique(41–42).” These two ideas combine in (C).

(A) While the art created in centers of style may bedistributed over a wide area, the example of the Konatésuggests that the “characteristics of a particular tribalstyle (39)” included in each mask make them far frominterchangeable.

(B) Artists from centers of style produce art for theneighboring tribes, but nothing in the passagesuggests that they encourage competition byinstructing other artists how to do so.

(D) sounds very close to the description of the Konatéfrom paragraph 3, but look closely: the “various tribes”within the geographical area are consumers of verysimilar art, not producers of it.

(E) A “diverse community of artists” doesn’t sound likeour example of a center of style, which is a singlefamily.

PrepTest 49 Explained

30

Page 33: LSAT_PT 49_Expl_web

Passage 3: Ancient female doctors

As usual, the Topic of this passage, surviving sourcesof information about women doctors in ancient Greeceand Rome, is introduced immediately. We learn inline 2 that these sources are “fragmentary,” but it isnot until lines 5–6 that the Scope of the passagebecomes clear. We’ll spend the rest of the passagelearning what the author thinks these surviving bits ofevidence tell us about the female doctors in ancienttimes. In another twist, the author tells us immediatelywhat she believes the surviving evidence proves: thatfemale medical professionals who were on a par withmale medical doctors existed in ancient times, whichpushes back the start date for the history of women inmedicine. This sounds a lot like her Main Idea. Whileit is rare that the Main Idea would show up in the firstparagraph of an LSAT passage, it is not unheard of,and it is clear from the Conclusion Keywords in lines6–7 (“the evidence shows that...”) and 10 (“So...”)that the author is putting forward her opinion.

The next paragraphs support the idea that the author’sconclusion appears early. Paragraph 2, paragraph 3,and paragraph 4 all discuss the evidence that,according to the author, shows that there were femalemedical doctors in ancient Greece and Rome.Paragraph 2 discusses the nature of the evidence, andthe fact that women doctors were apparently commonenough that they did not merit special notice.Paragraph 3 and paragraph 4 bring up two otheraspects of medical practice by women in ancient times:female doctors’ practice was not limited to midwifery,and female doctors were apparently considered withmale doctors indiscriminately. All three of theseparagraphs merely provide evidence for the conclusionexpressed in paragraph 1. The Purpose of the passagewill take this into account—it is to prove that womenwere medical doctors in ancient times.

Roadmap:

¶1—surviving info on women in medicine inancient times shows they were full doctors¶2—nature of surviving evidence, what thatshows¶3—female doctors, not just midwives¶4—similarity between male & female doctors

The Questions

14. (A) Global (Main Idea)

Even when the author’s Main Idea appears before theend of the passage, it must take the scope of theentire passage into account.

While the author might have been generous enough togive us her conclusion early on, that doesn’t mean wecan slack off for the rest of the passage. We’ve still gotto be sure we know how the remainder of the passagefigures into her argument, and we’ve got to make surethat the correct answer includes the scope of thoselater paragraphs. The only choice that accurately sumsup the scope of the entire passage while still providinga paraphrase of the author’s conclusion is (A)—the“range of textual evidence” in this choice is a crucialcomponent of the author’s main idea.

(B) focuses on the details about the writings of Plinythe Elder from the final paragraph, making this aclassic Faulty Use of Detail wrong answer choice.

(C) distorts the evidence as cited by the author. Sheemphasized the fact that ancient writings make nospecial comment on the existence of female doctors,showing that female doctors were not so rare as towarrant such notice.

(D) distorts the text. The author definitely argues thatfemale medical practitioners rose to the level of doctor,but not necessarily that they were also researchers.

(E) Scholars arguing that women did not practicemedicine in ancient times do not appear in thepassage—if they exist at all, perhaps the author willdeal with them in her next article.

15. (E) Detail

Never answer a Detail question on a hunch.

The answer to this question must be contained in thepassage. After all, we’re asked for something that theauthor mentions. We should take the time to skimthrough the text to make sure we’ve found the correctanswer to this Detail question. This research probablywon’t take too long, and it will pay off when we can becertain we’ve got the correct answer before moving on.Our research will lead us to (E); the author mentions inparagraph 4 that Pliny the Elder and other ancientwriters “quote the opinions and prescriptions of maleand female doctors indiscriminately. (53–54)”

(A) Most people reading this passage can probablythink of a few diseases that have become curable only

Section III: Reading Comprehension

31

Page 34: LSAT_PT 49_Expl_web

with the advent of modern medicine, but that doesn’tmean that the author of the passage mentions them.

(B) Paragraph 3 mentions evidence that some femaledoctors treated mainly female patients, but does so inthe course of showing “evidence of a broad scope ofpractice for women doctors (42–43)” in ancient times.

(C) Francesca de Romana is mentioned in paragraph 1as a candidate for the first female doctor, but a specificscholar advancing her candidacy is nowhere to befound.

(D) The training of medical doctors in ancient Greeceand Rome is outside the scope of the passage.

16. (A) Logic Function

Your Roadmap will help you answer questions on thepurpose of a paragraph.

According to our Roadmap, the third paragraph tells usthat female doctors in ancient times were more thanjust midwives; this information is additional evidencefor the Main Idea expressed in paragraph 1. Thisprovides us with a powerful pre-phrase of the answer,which is matched by (A).

(B) runs contrary to the author’s argument inparagraph 1. The argument in the first paragraph is theauthor’s own; why would she later argue that her ownconclusion is too broad?

(C) is too detailed. Paragraph 3 does mention inpassing some exceptions to the earlier conclusion—the female doctors who treated mainly femalepatients—but acknowledging their existence is not theprimary focus of the paragraph.

(D) and (E) paragraph 3 advances its own argument—that women docs weren’t just midwives—as evidencefor the conclusion formed in paragraph 1. It does notinclude anything that relates to both of the priorparagraphs.

17. (B) Inference

Anything added to the passage must conform to theauthor’s opinion.

You could be forgiven for assuming that this wassomehow a Global question. After all, it does ask us tocontinue the final paragraph of the passage, and mostof us are in the habit of placing the conclusion of ourarguments somewhere near the end of the passage.But the looser language (“could most logically beappended”) suggests that our task is Inference. We’lldo our research in the final paragraph and look for an

answer choice that logically continues its argument.We will, however, be careful to make sure that ourcorrect answer stays within the scope of the passage.

Paragraph 4 is focuses on “references in variousclassical works to…women’s writings on medicalsubjects (47–50).” The important fact about thesereferences, according to the author, is that they aremade in the same manner as references to the work ofmale doctors, without any special distinction on thebasis of the gender of the writer. Any continuation ofthis paragraph should further the author’s argument inthe paragraph. Only (B) does so, and even takes theextra step of connecting the argument in paragraph 4to the similar argument made in paragraph 2.

(A)’s strong phrase “only by” should make usskeptical, and that skepticism pays off: it contradictssome information in the final paragraph, which saysthat the references to women’s writing were made“without biographical information (57–58).”

(C) and (D) are also 180s, but they go further than (A)did and contradict the author’s earlier arguments.

(E) adds an element to the final paragraph thatundermines the argument already made, suggestingthat there is a “conflicting picture of ancient medicalpractice” rather than evidence for the uniformacceptance of women doctors.

18. (D) Inference (Author’s Attitude)

In Author’s Attitude questions, ask whether the attitudein question is positive or negative, and to what degree.Each answer choice in this question begins with anadjective, and—much like scanning the verbs in aPurpose question—we can scan the adjectives in anAttitude question. The “sources of information”mentioned in lines 1–5 are “fragmentary (2),” but theauthor argues that “even from these fragments we canpiece together a picture.” In fact, the informationcontained in these fragments, and the deductions madebased on that information, is the focus of the entirepassage. It sounds like the author is assuming that theinformation we can glean from these bits and pieces isaccurate, and that assumption is found in (D).

(A) The author is not “wary” of misinterpretation. Thereis nothing to suggest that the author believes that thefragmentary nature of the information makes it proneto misinterpretation.

(B) The author is not “optimistic” about lingeringquestions. It is hard to imagine a more completeanalysis than the author gives in her work, and thepassage doesn’t suggest that there are any “lingeringquestions” left unanswered.

PrepTest 49 Explained

32

Page 35: LSAT_PT 49_Expl_web

(C) The author is not “hopeful” about the sources’acceptance. This choice applies more to the author’sattitude towards her own conclusion than to theinformation she used to arrive at it.

(E) The author is not “convinced” of the sources’“appropriateness as test cases.” Additionally, a “newhistorical research methodology” is well outside thescope of the passage.

19. (D) Logic Function

Look out for different ways that the LSAT can phraseclassic question types.

While question 19 is not phrased like most of the LogicFunction questions that we’ve come across in ourpractice, it still asks us how a particular detail figuresinto the passage as a whole. Thankfully, once we’vefigured out what type of question we’re dealing with, wecan still use Kaplan’s tried-and-true strategies—in thiscase, digging into the context of a detail to determinehow it is used. The tribute in question is part ofparagraph 3, which focuses on the breadth of womendoctors’ practice in ancient times, and the fact that itwas not simply midwifery. This quote must have beenused to support the claim of the paragraph as a whole,which we find in (D).

(A) The only reference to “other doctors” comes inparagraph 4’s discussion of references to doctors’opinions in ancient medical works. These doctors arenever said to acknowledge each other.

(B) While the tribute quoted does mention one womandoctor’s “knowledge of medicine,” there is no evidencethat her knowledge was acquired through education.

(C) could be correct in a Detail question, but not in aLogic Function question. The epitaphs certainlysuggest that the women mentioned were effective, butthat implication is not the purpose of the statements’inclusion in the paragraph.

(E) makes an irrelevant comparison. The only tributesmentioned in this paragraph are for a female doctor—we have no tributes to male doctors for comparison.

20. (C) Inference

Scan the passage carefully to find support for thecorrect answer to an Inference question.

An Inference is something that must be true based onthe information in the passage. The support for it willnot necessarily be a lengthy citation of evidence or theconclusion of a paragraph. In fact, it is often the casethat an Inference is supported by only a single, easily

missed reference in the passage. When that is thecase, we must carefully research the passage to findthe correct answer, using the passage’s scope andRoadmap to guide our research.

(A) Our research turns up no references to how longwomen doctors practiced. Eliminate.

(B) The focus of the passage is squarely on womenwho were medical doctors—those who were notdoctors, and any informal medicine they may havepracticed, are outside the scope of the passage.Eliminate.

(C) Lines 13–16 refer to “Francesca de Romana’slicensure to practice general medicine.” According tothe passage, this took place in 1321, and was “theearliest known officially recorded occurrence of thissort.” This means that there must not be any knownofficial records of a licensed female doctor before1321—so no such records existed for women inancient Greece and Rome, which is stated almostdirectly in paragraph 2’s “There is no list of womendoctors in antiquity.” (C) must be true, and is correct.For the record:

(D) If anything, paragraph 3’s discussion of thedistinction between doctors and midwives suggeststhat there were some female doctors who acted asmidwives, although their practice was not limited tomidwifery. Eliminate.

(E) The only posthumous honors in the passage arethe epitaphs in paragraph 3, both of which are formedical—not civic—accomplishments. Eliminate.

Section III: Reading Comprehension

33

Page 36: LSAT_PT 49_Expl_web

Passage 4: Corn Productivity

While the Topic of this passage, maize, is immediatelyclear from the first sentence, the author hides the trueScope of the passage until the end of paragraph 1. Theparagraph meanders through several dif ferentelements before it arrives at the passage’s true focus.First, the author seems to focus on the effects ofmaize cultivation on societies, next he discusses the“primary reason (10)” for those effects—theproductivity of the crop. In these first few lines, itcertainly seems that we’re dealing with a Humanitiespassage about the effects of maize cultivation onancient societies. It is not until the Contrast Keyword“but” in line 15 that we find the true Scope: why ismaize so productive? We can also tell that the Purposeof this passage will be to answer that question. Theanswer forms the basis of the last two longparagraphs, and the depth of the technical informationin those paragraphs is typical of what we now know tobe a Natural Sciences passage.

As usual, the danger with this passage is getting boggeddown in the details, like the long description of theprocess of photosynthesis that opens paragraph 2. Thefirst detail worth a closer look is the introduction of theenzyme rubisco in line 27, and the author highlights ittwo separate ways: the Evidence Keyword “because,”and his evaluation of rubisco as “the most significantenzyme in the world (29–30).” But we also learn of aproblem with rubisco, the tendency for it to interfere withphotosynthesis under “many common atmosphericconditions (33).” Paragraph 3 gives us the solution tothat problem: a type of photosynthesis called “C-4photosynthesis (59)” that separates rubisco from theoxygen that can interfere with it. Once again, the lengthydetails of what this process involves are unimportant.The important thing is that the author has answered thequestion that he posed in the first paragraph, and theanswer is his Main Idea: C-4 photosynthesis isresponsible for the productivity of maize and severalother of the world’s most productive crops.

Roadmap:

¶1—Maize: why is it so productive?¶2—Process of photosynthesis. Rubiscoimportant but can interfere.¶3—C4 photosynthesis: prevents interference,leads to most productive plants.

The Questions:

21. (A) Global (Main Point)

Don’t let the details of the passage distract you fromthe author’s Main Idea.

With all of the details crammed into this passage, itwould be easy to lose sight of what the author is tryingto say. The structure of this passage is actually fairlysimple: the author asked a question at the end ofparagraph 1 and answered it by the end of paragraph3, and that answer is his Main Idea: C-4photosynthesis is responsible for the greaterproductivity of maize and several other crops. We canuse this answer as our pre-phrase, and we find a matchfor it in (A).

(B) brings the details from paragraph 1 back to theforefront, but these details really served more asexposition than the focus of the passage. Additionally,relative quality of the nutrients is not the focus of thepassage.

(C) again delves too deeply into the details, this timelooking too closely at paragraph 3.

(D) again focuses too deeply on the role of rubisco,plus it gets the details backwards. Rubisco’sinteraction with oxygen causes the problem that issolved by C-4 photosynthesis.

(E) distorts the details of the passage. There is onlyone mechanism discussed in the passage, and itprevents atmospheric gases from entering certain cellsin the leaf, not the leaves themselves.

22. (B) Global (Organization)

A question that deals with two long paragraphs isreally a Global question.

The scope of this passage didn’t really become clearuntil the end of paragraph 1, so a question about thelast two long paragraphs really includes the entirety ofthe author’s argument. That means we’ll treat thisquestion like a Global question about the organizationof the passage as a whole, but we do have oneadvantage: we can eliminate any answer choices thatbring in details from paragraph 1. Other than that, we’llwork through each choice to find the one that includesevery element of the last two paragraphs, withouteliminating or adding elements or going out of order.

(A) starts off with the cultivation of maize, an elementof paragraph 1. We can eliminate this one quickly.

(B) follows our Roadmap exactly. The “biochemical

PrepTest 49 Explained

34

Page 37: LSAT_PT 49_Expl_web

process” is photosynthesis, the “hindrance” isrubisco, and the “evolutionary solution” is C-4photosynthesis. (B) is the correct answer. For therecord:

(C) Not only does the problem inherent inphotosynthesis come later in the second paragraph,but the passage only provides one way that organismssolve that problem. Eliminate.

(D) Any cultural phenomenon associated with maize isonly found in paragraph 1. Eliminate.

(E) sounds good early on, but there is no suggestionthat the explanation of photosynthesis is “widely held”to be true—instead, it is presented as establishedscientific fact. Eliminate.

23. (B) Inference

Delve deeply into the details when—and only when—a question demands it.

Ideally, as we first read through this passage, we’dtreat the lengthy, technical details in paragraph 2 andparagraph 3 as a trap. We don’t really need tounderstand these details to understand the author’sargument, and their dense, scientific nature makes itdifficult and time-consuming for most examinees toslog through them. Besides, those details aren’t goingto get us any points, unless there’s a question thatrequires us to use them. But now we’ve arrived at justsuch a question, so we’ll start to delve.

Of course, that doesn’t mean we’ll blindly start readingthrough the details of the passage—we can still usethe question stem to guide our research. We’re lookingfor an evolutionary development that would give plantsan advantage similar to that enjoyed by C-4 plants, sowe’re looking for the details of how exactly C-4photosynthesis conveys its advantages. Our Roadmappoints us to paragraph 3, where the process of C-4photosynthesis is described. Not long into thatparagraph, line 45 signals that we’ve found what we’relooking for, “the key to that process.” C-4photosynthesis is so advantageous because oxygen,which interfered with normal photosynthesis by bindingwith rubisco, is excluded from the airtight cellscontaining the enzyme. We can conclude any otheradaptation that keeps oxygen away from rubisco willgive a species a similar advantage, and we find suchan adaptation in (B), which dispenses with rubiscoaltogether in favor of a less temperamental enzyme.

(A) is a 180. According to lines 47–48, the “bundlesheath cells” are the cells containing rubisco. If a plantseparated water into its constituent parts within those

cells, the rubisco would come into contact with moreoxygen, not less.

(C) Making the structures of the leaf impermeable tooxygen would help, but if they were impermeable tocarbon dioxide as well, the plant would have no fuelfrom which to build sugars. This adaptation wouldn’thelp.

(D) This adaptation distorts the process of C-4photosynthesis, in which “the bundle sheath cells[which contain rubisco, not the structures in whichwater is split] surround the vascular structures of theleaf (48–49).”

(E) would hinder photosynthesis, since rubisco is theenzyme which produces sugar by reacting with carbondioxide. A less reactive enzyme would necessarily beless effective.

24. (D) Logic Function

Logic Function questions are very similar to Role of aStatement questions from the Logical Reasoningsection.

How does the author’s reference to “all otheratmospheric gases” fit into the rest of the passage? Ithas to have some function, otherwise it wouldn’t bethere. The key to finding that function will be thecontext. With a bit of research, we find our answer justtwo sentences later. In lines 50–54, we learn that“carbon dioxide, which cannot enter these cells as agas, first undergoes a series of reactions to form anintermediary, nongas molecule named C-4.” That’s whyit’s important that the bundle sheath cells areimpermeable to all gases: it explains why the chemicaltransformation of carbon dioxide into C-4 is necessary.We find this in (D).

(A) There is no explanation given anywhere in thepassage for why certain atmospheric conditions causeexcess oxygen to build up.

(B) There is no claim advanced that other gases caninterfere with photosynthesis, either earlier in theparagraph or anywhere else in the passage.

(C) The conclusion that non-C-4 photosynthesis makesuse of atmospheric gases other than those used in C-4photosynthesis never appears in the passage.

(E) The passage makes no claim about oxygen levelsin C-4 plants.

Section III: Reading Comprehension

35

Page 38: LSAT_PT 49_Expl_web

25. (D) Inference

A quick skim of the answer choices can help directyour research for questions that lack clues in thequestion stem.

This is a straight-up Inference question which can oftenbe confounding. Without a hint as to where to directtheir research, many test-takers find themselvesskimming through the passage in the vain hope thatsomething will leap out at them. But when we skim theanswer choices for this question, we find that they areall statements about rice, which only appears once inthe passage, in line 60. The only thing we know aboutrice from the passage is that it is a C-4 plant like maizeand sugar cane, so our Inference must attribute one ormore of the properties of a C-4 plant to rice. (D) doesjust that with its assertion that rice sequesters rubiscoin bundle sheath cells, just like maize.

(A) is close enough to be quite confusing. In C-4plants, atmospheric gases cannot enter the cells whichcontain rubisco, not the cells in which water is split intoits constituent elements.

(B) Lines 20–24 tell us that “all plants split water intoits constituent elements…they use the resultanthydrogen to form one of the molecules they need forenergy, but the oxygen is released into theatmosphere.” There is no indication that the oxygencombines with anything else before it is released.

(C) is true, but it is outside the scope of the passage.Be careful not to use any of your outside knowledge inanswering the questions on the LSAT.

(E) How widely maize and rice are cultivated, or anycomparison between them other than the fact that theyare both C-4 plants, is outside the scope of thepassage.

26. (E) Inference

It is often helpful to consider the entire passage whenanswering an Inference question about the author’sopinion.

A skim of the answer choices reveals the commonelement of maize. Unfortunately, that probably won’tgive us much help in finding the correct answer, sincevirtually the entire passage deals with maize. Instead,we’ll have to approach this question like we wouldapproach a Logical Reasoning Inference question: we’llevaluate each answer choice based on the informationcontained in the passage itself. We’ll have to let eachchoice guide our research.(A) reverses the central premise of paragraph 1, that

maize reshaped several cultures because of itsproductivity. Eliminate.(B) According to lines 20–24, “all plants” releaseoxygen into the atmosphere as a by-product ofphotosynthesis. Maize is no exception. Eliminate.(C) distorts the thrust of the entire passage. Theauthor attributes maize’s productivity entirely to its useof C-4 photosynthesis, not any other element.Eliminate.(D) There is evidence in paragraph 1 that Europeanculture was changed by the introduction of maize, butno evidence that maize cultivation required any specialtechniques that the Europeans lacked. Eliminate.(E) pieces together elements from the first and lastparagraphs of the passage. C-4 photosynthesis isindeed an evolutionary adaptation to more effectivelyutilize rubisco, and humans have indeed benefitedfrom the resultant higher productivity. (E) must be true,and is correct.

27. (A) Inference

The answer to an Inference question will oftencombine different elements of the passage.

Once again, we’re faced with an Inference questionthat refuses to point us to a particular part of thepassage. This time, rubisco is the element that’scommon to all five of the answer choices, but just likein the last question, rubisco is common enough in thepassage that it will be difficult to pin down any area forresearch. Thankfully, we can focus our researchingefforts on paragraph 3 and the latter half of paragraph2 as we work through the choices:(A) quickly turns out to be the correct answer. In C-4plants, rubisco is isolated in airtight structures withinthe leaf, the “bundle sheath cells” of line 48. But innon-C-4 plants, oxygen can reach the rubisco and bindwith it, ”interfering with the photosynthetic reaction(35),” so the rubisco must not be in an airtightstructure. For the record:(B) C-4, and not rubisco, is named “for the four carbonatoms it contains (53–54).” Eliminate.(C) We learn in paragraph 2 that rubisco is part of thephotosynthetic reaction that converts carbon dioxideinto sugars, but we only know that “rubisco assists inthe sugar-forming chemical reaction (27–28),” not thatit is necessary to that reaction. Eliminate.(D) Rubisco is responsible for the detrimental effectsof oxygen buildup in all plants—it is the bundle sheathcells of maize and other C-4 plants that prevent thosedetrimental effects. Eliminate.(E) There is no information given in the passage aboutoptimizing the C-4 process. Eliminate.

PrepTest 49 Explained

36

Page 39: LSAT_PT 49_Expl_web

SECTION IVLOGICAL REASONING

1. (C) Flaw

Remember, Flaw questions are closely related toAssumption questions.

Arguing over unemployment statistics is something ofa pastime for economists and politicians alike. Now itseems that the editors of a local paper have gotten intothe act, concluding that unemployment in their city isgetting better. Their reasoning is based on a singlepiece of evidence: studies showing that the number ofunemployed people looking for work has decreased.

Notice the difference between the terms of theevidence and the terms of the conclusion: theconclusion deals with unemployment in general, whilethe evidence only looks at unemployed people who arelooking for work. The editorialist has assumed that all(or perhaps most) unemployed people are activelylooking for jobs—this would mean that the decreasingnumber of unemployed looking for jobs must mean adecreasing total number of unemployed. (C) points outthe flaw based on this assumption. If many of theunemployed workers have stopped looking for jobs, theassumption, and the conclusion based on it, would fallapart.

(A) The government’s responsibility, or lack thereof, isentirely outside the scope of the editor’s argument.

(B) The studies about the next two years may be tooshort to justify a conclusion about a general trend, butthe argument’s conclusion only deals with the period inthe studies—no flaw there.

(D) Neither governmental effor ts to reduceunemployment nor high-paying jobs are relevant in theeditorial.

(E) Other economic indicators are also outside thescope of the argument.

2. (B) Weaken the Argument

Be on the lookout for variations on common questiontypes on the LSAT.

This question stem may have thrown you for a loop, butit’s actually not anything new. This type of stem wascommonly seen in Weaken questions on the first fewreleased LSATs, but hadn’t made an appearance forquite some time prior to PrepTest 49. Whether or notyou had seen this type of stem in your practice, youcould have figured out what it was looking for. After all,

“evaluating” an argument is really determining itsstrength or weakness.

This argument extols the virtues of garlic at reducingthe risk of cardiovascular disease by lowering levels ofcholesterol and triglycerides in the blood. Theevidence, presented by the not-so-subtle Keyword“evidence,” is a study of two groups of patients. Thefirst group took a garlic supplement and showed amuch greater reduction in cholesterol and triglyceridesthan a second group, which only took a placebo. Butwhat else did the two groups do? The argumentdoesn’t tell us anything about the rest of their diets,any exercise program, or any of a number of things thatcould also have affected their cholesterol andtriglyceride levels. It simply assumes that all of theseother factors don’t matter, a serious omission. That isthe key to evaluating this argument—thinking aboutalternative reasons for the study’s findings. (B) givesus one such alternative to think about: the diets of thetwo groups. Looking at alternative possibilities is oftenimportant in Strengthen/ Weaken questions.

(A) The availability of the garlic tablets is irrelevant totheir efficacy.

(C) The argument provides no evidence that the four-month study period was significant in the effectivenessof the garlic—be careful not to focus on insignificantdetails like this one.

(D) We are given no evidence that the garlic tablet usedin the study constitutes a “large amount” of garlic, noris the tolerance of people for garlic relevant to how wellit reduces cholesterol.

(E) The tablet manufacturer’s advertising has noimpact on the strength of the argument.

3. (D) Inference

Evaluate the choices in Inference questions byrelating them to the stimulus.

Don’t be fooled by the “if” in the first sentence. Thisquestion doesn’t require any formal logic. Instead, theeducator bemoans the loss of quality in today’seducation, telling us how the emphasis on degrees hasopened the door to obtaining meaningless credentialsby completing courses without leaning anything. TheKaplan Method for Inference questions tells us not tospend time predicting, so we’ll move straight into thechoices:

(A) The author tells us what has to happen forcredentials to be meaningless, but never says thatmore meaningless credentials are being granted.Eliminate.

Section IV: Logical Reasoning

37

Page 40: LSAT_PT 49_Expl_web

(B) Again, the educator doesn’t claim that it’s noweasier for students to get a meaningless degree. Heonly tells us that it’s possible to do so. Eliminate.

(C) is far too extreme. The educator warns of thedanger of a particular type of degree or certificate, butdoesn’t advocate the abolishment of all degrees andcertificates. Eliminate.

(D) Well, if it’s possible to obtain a degree “withoutever learning much of value,” as the author says, thenit’s cer tainly true that a degree alone doesn’tguarantee someone knows anything worthwhile. (D)must be true, and is correct. We can quickly eliminatethe last choice:

(E) The effort invested to obtain a degree doesn’tnecessarily relate to the benefits it bestows. After all,“plodding through courses” sounds like an effort, butit leads to what the author calls a meaningless degree.

4. (E) Strengthen / EXCEPT

Characterize the choices carefully in EXCEPTquestions.

The four wrong answer choices all strengthen theargument, while the right answer to this question eitherweakens it or does nothing. The argument itself warnsus that tabloid journalism that focuses on politicians’private lives causes several problems: talented peopledon’t pursue political careers in order to protect theirprivacy; reporters focus on character flaws instead ofthe issues; and the journalism itself is “trivial.” Inshort, the essayist assumes the politicians’ livesaren’t pressworthy.

The essayist certainly makes press coverage ofpoliticians’ private lives sound awful. Four of theanswer choices will strengthen that reasoning, eitherby bolstering some of the evidence already given or bygiving more evidence against tabloid journalism. Thecorrect choice will either be irrelevant, or will give us areason why this type of journalism is actually a goodthing. Let’s skim the choices together:

(A) strengthens the argument against press coverageof private lives—if the coverage is hideously inaccurateas well as intrusive, it’s just that much worse.Eliminate.

(B) provides another reason why the journalism istrivial: it distracts from the real issues in a campaign.Eliminate.

(C) Now the reporting on politicians’ private lives iseven worse. Much of it isn’t even true! It’s just a bunchof rumors circulated by the opposition. Eliminate.

(D) bolsters the evidence that press coverage ofpoliticians’ private lives keeps good people out ofpolitics. Eliminate.

(E) Ah-ha! Here’s a reason why coverage of politicians’personality flaws could actually be a good thing—itaffects their job per formance. (E) weakens theargument, so it’s our correct answer.

5. (C) Inference (Formal Logic)

Inference questions don’t have to ask for what mustbe true; they can ask you to find what could or cannotbe true as well.

This atypical Inference question tells us to look for achoice that CANNOT be true. If the correct answer isimpossible based on the statements in the stimulus,then the four wrong choices will be possible—in otherwords, they all could be true. We’ll keep that in mindwhen we evaluate the answer choices.

“Most” is a clue that formal logic will play a role in aquestion, just like “if” or “only if.” Here, we find thatmost veterinarians (meaning at least half of allveterinarians) have a strong interest in biologicalscience. Most veterinarians also choose theirprofession primarily because of their love for animals.

Be careful when there are two “most” statementsabout a single group. While there must be someoverlap between the two characteristics—here, theremust be some veterinarians that have a strong interestin biology and also chose their profession because oftheir love for animals—the overlap doesn’t have toinclude the majority of the group. This concept is oftentested on the LSAT.

Finally, we hear about “people who are seriouslyinterested in biological science but lack any speciallove for animals.” This group includes no prominentveterinarians. Note the qualifier “prominent” In theargument’s most definite statement. This last groupcould include some veterinarians without violatingeither of the first two statements, as long as thoseveterinarians aren’t prominent ones. Armed with all ofthis information, let’s test the choices:

(A) could be true. Even those vets that love animalsand chose their profession on the basis of that lovecould still love biological sciences even more.Eliminate.

(B) is possible. The two characteristics of most vetscould overlap in a majority of vets, even though theydon’t have to. Eliminate this choice, but note: if thisquestion had asked for what must be true, (B) wouldbe a particularly dangerous (and common) trap.

PrepTest 49 Explained

38

Page 41: LSAT_PT 49_Expl_web

(C) directly contradicts the last sentence. We knowthat there are no prominent veterinarians among thepeople who are interested in biological science butdon’t love animals. (C) cannot be true, and is correct.For the record:

(D) could be true. The vets at university researchcenters don’t have to be part of the majority of vetswho chose their profession because they love animals.(D) is another trap that deals with the overlap betweenthe two different majorities of veterinarians. Eliminate.

(E) Vets who aren’t prominent and what they considerimportant for success are way outside the scope of theargument.

6. (A) Flaw

An argument is flawed when the evidence fails toestablish the conclusion.

Somehow this question manages to discuss mythicalhalf-horse, half-human creatures without ever using theword “centaur.” But it does point out an interesting factabout the myths: in all of the ancient cultures that hadsuch a myth, centaurs were seen as violent andsavage, in contrast to the traditional view of horses asgentle and noble. Since human cultures often usemyths to express unconscious thoughts, the authorclaims that the mythical depiction of centaurs reflectpeople’s unconscious fear of the horse, assumingcorrelation from evidence that merely suggestscoincidence.

Well, we have to take the evidence at face value, so wehave to accept that myths represent unconsciousthoughts. But why does the depiction of centaurs haveto reflect an unconscious fear of the horse? Whycouldn’t it represent, say, an unconscious fear of thepeople riding the horses? The author doesn’t really giveus a reason why there’s only one possibleinterpretation of the centaur myth, he just assumesthat his explanation is the only possible one. (A) picksup on this faulty assumption—the author doesn’t everestablish that centaurs represented horses in people’sminds.

(B) The fear of horses is what the author fails toestablish, not the validity of people’s reason for thatfear.

(C) The expression of unconscious thoughts ismentioned in the last sentence as the use for myths,but the suppression of unconscious thoughts isoutside the scope. Don’t fall for an answer choice justbecause it’s confusing!

(D) Even if the myth was borrowed from one of the

ancient cultures, the fear behind it could still be real,and that fear is the author’s concern.

(E) Always take the evidence at face value on the LSAT.If the evidence states that people use myth for theexpression of unconscious thoughts, the authordoesn’t need to establish the truth of that statementany further.

7. (C) Main Point

Don’t let the details of the argument distract youfrom the author’s main conclusion.

A construction like “there would seem to be” usuallysignals that the author is going to disagree withwhatever statement seems to be true. This argumentis no exception. After stating that there seems to belittle danger from the chemicals in treated lumber, theauthor suggests that we should still test the safety ofthese chemicals, since there’s a chance thatconsumers could ingest them. Don’t worry too muchabout the examples of consumers ingesting thechemicals—the Keyword “since” has already shown usthe dividing line between the author’s conclusion andhis evidence. The conclusion is that we need to testthe chemicals in treated lumber, restated in (C).

(A) misses the author’s qualification of the firstsentence. This sentence wasn’t the conclusion, it wasa point that the author disputes in order to make hisconclusion.

(B) is a 180 of one of the details of the argument. Theauthor states in the first sentence that treated lumberis safer because it “is used outdoors where fumescannot accumulate.”

(D) focuses on a detail from the last sentence.

(E) is too extreme. The author never says treatedlumber is definitely dangerous, only that the possibledangers should be studied.

8. (A) Weaken the Argument

Keep a close eye on the scope of the argument.

We’re asked here to check the reliability of a methodfor determining where an ancient relic has been. Thatmethod involves analyzing the pollen left on the relic,which the author concludes is “one good clue” as towhere the relic has been, since certain pollens comefrom plants that are known to be unique to certainareas.

Did you catch the scope shift in the evidence? Justbecause the plants are unique to certain areas does

Section IV: Logical Reasoning

39

Page 42: LSAT_PT 49_Expl_web

not mean that the pollens are also unique to certainareas. Perhaps the pollen is moved around by a naturalprocess or by human intervention. If that’s the case,then identifying the pollen on an artifact wouldn’tnecessarily pin it down to a specific location, as (A)suggests.

(B) is an irrelevant comparison between methods ofdetermining the history of movement of a relic. Pollenanalysis could still be a reliable method, even if othermethods are less complicated.

(C) and (D) could easily be tempting answers, sincethey both seem to suggest that pollens either weren’tunique to certain regions or aren’t a useful tool in thefirst place. But the method in question examinespollens from plants “known” to be “unique to certainareas,” so pollens from several areas or pollenslacking geographical distribution data wouldn’t be usedby the method at all.

(E) The expense and difficulty of the method don’taffect its reliability.

9. (E) Principle

Pre-phrase the answer to a Principle question byfinding a general, law-like rule that would apply to thesituation in the stimulus.

The marketing executive who wrote this statementmust not have been paying attention to the recentexplosion of herbal supplements for sale. He actuallydid some research on the medical efficacy of herbaltinctures before promoting and distributing them, andcame across evidence that they were not medicallyeffective. So he concludes that marketing herbalremedies would produce the results he intended—inother words, they wouldn’t “add to our profits.” We cansum up this entire argument quite simply by sayingsomething like, “Marketing a new product will not beprofitable if reliable authorities say the product isineffective.” If your pre-phrase was at all close to thissummation, you no doubt found (E) to match it almostword for word.

(A) The executive did consult a number of “reliablemedical publications,” but what exactly makes thesepublications effective is outside the scope of hisargument.

(B) was probably tempting, since the executive seemsto believe all the research showing herbal tinctures tobe ineffective. But his argument deals specifically withthe profits to be gleaned from such products, so theprinciple guiding the argument must do so as well.

(C) contradicts the executive’s line of reasoning as it

relates to the medical publications. Besides, he didn’tconsult these publications by themselves, he alsorelied on his advisory staff.

(D) is too extreme. There is no evidence that herbaltinctures have adverse effects, only that they areineffective.

10. (D) Inference (Formal Logic)

In Formal Logic questions, beware of answer choicesthat confuse necessity and sufficiency.

You may not have immediately noticed the Formal Logicin this question, since it does not appear in the classicif/then formulation. But you should learn to recognize thelanguage of necessity: anything that “must” happen inorder for something else to occur is the result in a FormalLogic statement. Thus, we can translate the firstsentence of this stimulus as, “If an artwork is great, thenthat artwork expresses a deep emotion.” Remember toform the contrapositive: “If an artwork does not expressa deep emotion, then it is not a great work of art.”

The double negative in the second sentence cancelsitself out, so we get, “If an artwork expresses anemotion, then the artwork’s creator must be capable ofexperiencing that emotion.” The contrapositive wouldstate that, “If an artwork’s creator is incapable ofexperiencing an emotion, then their work cannot expressthat emotion.”

As in most Inference questions that involve Formal Logic,we can combine the statements once we have translatedthem. Here, the stimulus tells us that an artwork’screator must be capable of experiencing any emotionexpressed by their work, and so the creator of a greatwork of art must be capable of experiencing the deepemotion necessary to such an artwork. Let’s use thisnew statement to evaluate the choices:

(A) was probably tempting, since we think of computersas incapable of experiencing emotion. But notice that thestimulus only refers to the “capacity to” experienceemotion, while (A) insists that the actual experience ofthe emotion is necessary. That’s why (A) does not haveto be true, and can be eliminated.

(B) does not have to be true. While we know that greatart must express deep emotion, the stimulus nevermakes the correlation between depth of emotion andgreatness of art that (B) does. Eliminate.

(C)’s logic is backward, and so confuses necessity andsufficiency. The stimulus says that the expression ofdeep emotion is a necessary quality of great art, but (C)claims that such an expression is sufficient to consideran artwork great.

PrepTest 49 Explained

40

Page 43: LSAT_PT 49_Expl_web

Don’t be fooled by the introduction of computers in (D).The choice focuses on computers’ inability toexperience emotion and how it relates to their ability toproduce great ar t. Sure, if a computer cannotexperience emotion, then according to the logic of thestimulus, they cannot create art that expresses anyemotion, and thus computer-created art cannot begreat. (D) must be true, and is correct.

(E) Watch out for the distinction between great artworkand a great artist. No connection between the two ismade in the stimulus, so (E) is out of scope.

11. (A) Paradox

The correct answer to a Paradox question must showhow the apparently conflicting elements of thestimulus can occur together.

With all of the car commercials touting antilock brakes,it would be easy to think that they’re a feature every carshould have, but the consumer activist thinks otherwise.Even though she admits that antilock brakes havereduced the incidence of multiple-car collisions, shebelieves that auto manufacturers should stop equippingcars with the brake systems “to save lives.” Therein liesthe problem. It doesn’t seem to make much sense thatantilock brakes reduce the incidence of certain types ofcar accidents and yet removing them would save lives.But that’s the point of this question. Our job is to findthe answer choice that explains how these conflictingstatements can both be true—how antilock brakes canboth prevent accidents and result in more fatalities. (A)gives us this explanation. If people in cars with antilockbrakes don’t wear their seatbelts, then they are morelikely to die in any accidents that do occur.

(B) and (D) give us additional reasons to removeantilock brakes in favor of traditional brakes, but neitherone explains how antilock brakes lead to greaterfatalities in auto accidents.

If anything, (C) argues that antilock brakes shouldn’t beremoved. If inexperienced drivers (the most dangerousdrivers on the road) find antilock brakes easier to usethan traditional brakes, then antilock brakes make thesedrivers safer.

(E) is an irrelevant comparison between the effects ofantilock brakes on the incidence of different kinds ofaccidents. In fact, we know from the stimulus thatantilock brakes reduce the incidence of multiple-caraccidents, so if (E) were true, the brakes would lead tofewer other accidents as well, arguing against theactivist’s recommendation.

12. (A) Flaw (Formal Logic)

Don’t let any outside knowledge creep into yourevaluation of the stimulus.

No matter how much you may have heard about oilcompany profits in the news or at the gas pump lately,you shouldn’t let that information color your evaluationof the politician’s argument. On the LSAT, you have toact like the stimulus is the only information you have,just like you would have to use only the permissibleevidence as a lawyer in a court case.

Now, this politician argues that regulations designed toprevent collusion in the oil industry don’t need to betightened—in fact, that they are excessive. How doeshe arrive at this conclusion? Well, he actually uses afairly simple bit of formal logic, even though he cloaksthe logic in lots of complicated language about profitsand burdensome regulations. Here’s how it breaksdown: if the regulations are not excessivelyburdensome, then the oil companies will make profitssufficient to motivate their risky exploration. Quick,form the contrapositive: if the oil companies don’tmake profits that are sufficient to justify their risk, thenthe regulations on them are excessively burdensome.

If this argument weren’t flawed, we’d expect to see thislogic statement linked to another one, especially sincethe politician’s conclusion is the result of his logicstatement’s contrapositive: the regulations areexcessively burdensome. Instead, we get a piece ofinformation that doesn’t seem to connect: oil industryprofits aren’t the highest among all industries. Thepolitician is making an assumption to link this newterm with his Formal Logic statement: if the oilindustry’s profits are sufficient to justify their risks,then the profits must be the highest among allindustries. But he’s not allowed to make an unjustifiedassumption, so this is the flaw in his argument, as (A)points out.

(B) is an easy trap, especially if you allowed outsideknowledge (or opinions) about oil companies to creepin. But the politician never attacks the character of theoil companies—he’s too busy defending them fromregulations.

(C) Causal arguments and correlations are welloutside the scope of this argument.

(D) The politician mentions evidence that the oilindustry’s profits aren’t the highest among allindustries: “recent data.” Thus, he can’t have doneanything based on an absence of evidence about oilindustry profits.

Section IV: Logical Reasoning

41

Page 44: LSAT_PT 49_Expl_web

(E) is way outside the scope—the politician draws aconclusion about the oil industry in general, but it isbased on evidence about that industry, not anunrelated or atypical example.

13. (B) Assumption

The author’s assumption must connect anymismatched terms in the conclusion and theevidence.

This question falls in the part of the Logical Reasoningsection that is usually dominated by difficult questions,but Kaplan’s approach to Assumption questionsmakes it a snap. The author concludes that modernsculptures are monochromatic due to amisunderstanding about ancient sculpture. Peopleonce believed that ancient sculptures were uncolored,but we now know that the ancient sculptures had beencarefully painted. It shouldn’t take too much work tonotice the mismatched terms in the evidence and theconclusion: the evidence is all about ancient sculpture,and the conclusion deals with modern sculpture. Theauthor has to assume that our knowledge of ancientsculpture influenced modern sculpture, (B).

(A) the natural beauty of the materials used in modernsculpture is outside the scope of the argument, whichis entirely concerned with whether or not the sculpturesare colored.

(C) compares the susceptibility of modern and ancientsculpture to moisture damage. But the only moisturedamage mentioned in the stimulus was to the paint onancient sculptures; since modern sculptures aren’tpainted, there’s no comparable possibility of suchdamage.

(D) Ancient paintings are outside the scope of theargument, which deals only with sculpture.

(E) might be the next step if the argument’s conclusionis correct, but we’re looking for the assumption withinthe argument, not an inference based on it.

14. (E) Inference

Every LSAT question comes with one correct answerchoice and four terrible ones.

Don’t be fooled by a stimulus that asks you whichanswer choice is “most strongly supported.” The LSATwill never give you a set of answer choices where onechoice is very strongly supported, another choice issomewhat supported, two more of the choices have alittle bit of support, and the last choice isn’t supportedat all. The correct answer to any Inference question is

supported by the stimulus (it must be true) and theincorrect choices are not supported by the stimulus atall (they could be or must be false).

This stimulus gives us an insight into the cockpits ofnewer and older commercial airplanes. The olderplanes were designed so that all of the crew memberscould immediately view any changes in the controlsmade by any one of the crew members, but the newerplanes are designed differently. In the new planes, it ismore difficult for the entire crew to see one member’schanges to the controls, so the flight crews have to talkto each other more often about the control changesthey make. Using that information, we can evaluate thechoices to see which one must be true.

(A) sounds good up until it says the frequency of verbalcommunication depends on “how long it takes toperform those changes” in the flight control settings.The length of time it takes to make the changes isoutside the scope of the argument. Eliminate.

(B) Calling verbal exchanges of information “the mostvaluable means available for performing cross-checks”is much more extreme than the stimulus. Just becausea certain course of action is necessary doesn’t meanit’s the best course of action available. Eliminate.

(C) is also more extreme than the stimulus. There’s noevidence that crews in older airplanes had absolutelyno need to discuss flight control changes, only thatcrews of new airplanes must discuss such changes“more frequently” than crews of older airplanes.Eliminate.

(D) The stimulus says that flight control changes inrecently manufactured aircraft “are harder to observe,”not that they are impossible to observe. Once again,(D) is more extreme than the stimulus and can beeliminated.

(E) Finally, something that must be true based on thestimulus. The “other means for performing cross-checks” in older aircraft was the control panel design,which allowed each member of the flight crew to seeanother member’s changes to the controls. That“routine” means for performing cross-checks wasremoved in the newer aircraft, leading to an increase inverbal communication as an alternative. (E) is correct.

15. (A) Method of Argument (Formal Logic)

The correct answer to a Method of Argumentquestion will be a 1:1 matchup between the stimulusand the answer choice.

This question, which deals with a proposed FactorySafety Act, includes the most obvious Formal Logic

PrepTest 49 Explained

42

Page 45: LSAT_PT 49_Expl_web

statement of the entire section, with an “only-if” in thefirst sentence. We know from translating thisstatement that if a company operates an auto factory,it must register that facility as a “class B” factory. Thesecond statement tells us that class B factories musthave punctual inspections, and the argumentconcludes that the Factory Safety Act would preventauto factories from postponing inspections. This is aclassic Formal Logic formulation: If A, then B; If B, thenC, therefore, if A, then C. That’s what we’ll look for inthe choices, and we find it in (A): two provisions of theFactory Safety act combine to produce a certain result.

(B) There’s only one interpretation of the FactorySafety Act given in the stimulus, not two.

(C) No existing legislation is mentioned in thestimulus.

(D) The two provisions of the proposed Factory SafetyAct don’t conflict—in fact, they combine to produce acertain result.

(E) There’s no analogy in the stimulus. The provision inquestion is only discussed as it relates to a singlespecific situation.

16. (A) Assumption (Formal Logic)

The author’s assumption must be true in order for theconclusion of the argument to be true.

This argument sounds like it wants to refute Keats’“Ode on a Grecian Urn.” The author’s conclusion is justthe opposite of the poet’s famous final couplet: thereis, in fact, a difference between beauty and truth. Withan argument about such abstract topics, it’s easy toget confused about what exactly the evidence is sayingand how it relates to the conclusion. But the KaplanMethod for Assumption questions, finding and fillingthe gap between the evidence and the conclusion, willstill serve us well. Keep a close eye on any terms thatonly appear in either the evidence or the conclusion,and try to link them.

The evidence begins with a fairly complicated FormalLogic statement, which we can translate as, “If there’sno difference between beauty and truth, then the mostrealistic artworks are the best,” or, “If B = T � mostreal art = best art.” You should also quickly form thecontrapositive: if the most realistic art is not the bestart, then beauty and truth aren’t the same thing (ifmost real art ≠ best art � B ≠ T). This statement actsalmost like a secondary conclusion, and we get a hintthat we should treat it as such from the Keyword“since” that follows it. So the author believes that thisFormal Logic statement is true because the most

realistic artworks are also the most truthful artworks.Well, if that’s the case, then they’d also be the mostbeautiful works of art; after all, this statementoperates under the premise that beauty and truth arethe same thing. So the most realistic artworks are themost truthful, which means they’re also the mostbeautiful. Why would that necessarily mean they alsohave to be the best? It wouldn’t, unless the mostbeautiful artworks are also the best. (Note the two-stepshift in that assumption: one from beauty to mostbeautiful, a second from most beautiful to best.) That’sthe final, missing piece of this argument, and theauthor’s assumption, which we find in (A). And asusual, we could’ve gotten halfway to the correct answersimply by noticing that “the best artworks” appears inthe evidence, but not the conclusion.

(B) might be implied by the statement that the mostrealistic artworks are also the most truthful, but it isnot necessary for the author’s conclusion to be true.

(C) is too extreme. The argument states that many ofthe most realistic artworks are not among the bestartworks, but (C) implies that all realistic artworksaren’t among the best.

(D) is tempting, but the argument deals with artworksthat are superlative—the most realistic, the mosttruthful, the most beautiful. Artworks that are simplybeautiful are outside the scope.

(E) The inherent subjectivity of artwork is also outsidethe scope of the author’s argument.

17. (A) Parallel Reasoning

Familiarity with the causal argument structures arehelpful in all parts of the Logical Reasoning sections.

When an argument uses evidence of a correlationbetween two things (X and Y happen together) toconclude that one caused the other (X causes Y), youshould immediately think of three possiblealternatives: the opposite is true (Y caused X), somethird factor caused one or both (Z caused Y and/or X),or the connection between the two is just acoincidence. This question actually uses one of thesethree alternatives as its conclusion. Instead of usingthe correlation between studying music and proficiencyat mathematics to argue for a causal relationshipbetween the two, the author concludes that a thirdfactor (an encouraging family) is just as likely to causeboth. If we use this basic structure—X didn’tnecessarily cause Y, because Z could have caused Xand Y—to analyze the choices, (A) immediately leapsout as parallel.

Section IV: Logical Reasoning

43

Page 46: LSAT_PT 49_Expl_web

(B) starts off well, with evidence about a correlation,but the conclusion veers off onto the superiority ofsecondary schools, a topic that has nothing to do witha causal relationship.

(C) uses a different approach to arguing against acausal relationship than the stimulus, claiming that therelation between to factors is just a coincidence.

(D) contains several scope shifts that the originalargument does not. For example, it shifts from a groupthat is “required to study” biology and chemistry to“those who have mastered” the two subjects.

(E) again draws a conclusion against a causalrelationship, but it does so by suggesting the causalfactor could be effective in smaller amounts—that lessvigorous exercise could be as effective as vigorousexercise.

18. (D) Assumption

Use Keywords to nail down evidence and conclusionin arguments with lots of filler and backgroundinformation.

Medical ethics are a particularly thorny territory, andone that is increasingly occupied by lawyers. Thisstimulus concerns itself with one aspect of thoseethics, the potential conflict between the physician’sduty to see to their patient’s well-being and thepatient’s right to know about their health. According tothe stimulus, the patient’s right should prevail. It’seasy to find the evidence for this conclusion, becausethere’s only one small piece of it and it comes after theKeyword “since”: the patient’s right is a basic right.Unfortunately, that fact alone doesn’t prove theconclusion is true, unless the author also assumesthat basic rights should win out in any conflict. (D)rephrases this assumption—if people’s basic rightsshould never be violated, then certainly those rightswould win out over other duties. Don’t be turned off bythe strong word “never” in this choice: we’re looking foran assumption that makes the conclusion “followlogically,” and this one does.

(A) may be true, but this right isn’t touched on in thestimulus, so it can’t affect the conclusion.

(B) Actions are also outside the scope of the stimulus,which is only concerned with rights and duties.

(C) traps the unwary with a red herring. The lastsentence implies that objects do not have rights, butthat fact doesn’t get us any closer to connecting theevidence and the conclusion.

(E) is also tempting, since it certainly makes it more

likely that the patient’s basic right shouldn’t beviolated. But it leaves open the possibility that aphysician’s duty is stronger than a patient’s basic right,which would disprove the conclusion.

19. (A) Inference (Formal Logic)

Be sure to translate and combine any Formal Logicstatements.

It is often the case that the answer to an Inferencequestion comes from a single sentence in thestimulus, or even a fragment of a sentence, and thisquestion is no exception. We learn in this stimulus thatmost of the world’s forests have fragmentedecosystems and cannot sustain themselves in the longterm, but harbor many endangered species. The finalsentence’s “requires” clues us into a Formal Logicstatement: If a fragmented forest is to maintain all ofits plant and animal species, then resource managersmust regularly intervene. The contrapositive tells usthat if resource managers do not intervene regularly,then the fragmented forests will lose at least some oftheir plant or animal species. Using these facts, wecan evaluate the choices:

(A) is a perfect combination of two facts in thestimulus: most of the world’s forests are fragmented,and fragmented forests will lose some species withoutintervention. (A) must be true, and is correct. For therecord:

(B) is a bit too specific. We know that fragmentedforests harbor the world’s most endangered species,and these forests will lose some species if resourcemanagers do not intervene, but the most endangeredspecies wouldn’t necessarily be the ones lost.

(C) is a distortion of the facts in the stimulus. Afragmented forest cannot sustain itself, regardless ofwhether it loses any more species.

(D) and (E) Complete, fully functioning ecosystems areoutside the scope of the argument, as are the placeswhere resource managers currently intervene.

20. (D) Assumption

A shift in scope over the course of an argumentusually signals an assumption.

Any information about ADD will make the lives ofteachers everywhere much easier. This magazinearticle gives us one new insight about the disorder. Itconcludes that ADD in kids may be exacerbated byconsumption of sugar. The evidence deals with theeffect of sugar consumption on adrenaline—

PrepTest 49 Explained

44

Page 47: LSAT_PT 49_Expl_web

consuming large amounts of sugar leads children toproduce large amounts of adrenaline. If you noticedthat the evidence talks about adrenaline instead ofADD, you’ve already found the assumption. We can putthese together to make a strong pre-phrase: moreadrenaline means worse ADD in children. We find amatch for this pre-phrase in (D).

(A) and (C) Children who don’t have ADD andtreatment of ADD are outside the scope of theargument.

(B) is too extreme. The argument focuses on a factorthat may make ADD worse, not a factor that causesADD outright.

(E) contradicts the stimulus. While the adrenalineproduction is “especially noticeable” if sugar isconsumed without food, the increase in adrenalineoccurs based on the consumption of a lot of sugar.

21. (C) Principle (Formal Logic)

Don’t let confusing language in the answer choicesthrow you off.

The ethicist’s argument centers around what shouldand should not be the basis for praising a person’sabstinence from alcohol. He argues that those whoabstain due to a lack of financial means shouldn’t bepraised, and that those who abstain due to a lack ofdesire shouldn’t be praised either. But the laststatement is part of a not/unless Formal Logicformulation, so we can translate it: if someone whoabstains from alcohol because they lack the desire topartake should be praised, then their lack of desiremust result from discipline. Conversely (orcontrapositively), if the lack of desire isn’t the result ofdiscipline, then it isn’t worthy of praise. To make thisinto a principle, all we have to do is make the specificsof the argument into generalities. For example, insteadof focusing on the specific behavior of avoidingalcoholic beverages, we’ll look for answer choices thatdeal with behavior in general and whether or not it isworthy of praise. We find one in (C), which takes intoaccount both the financial considerations of the firstsentence and the disciplinary considerations of thelast sentence.

(A) Both the consequences and the social context ofbehavior are outside the scope of the argument.

(B) and (E) Blame for an action is also outside thescope of the argument, which only deals with praise.

(D) may in fact be the case, but the ethicist isn’tconcerned with how arduous the process of acquiringself-discipline is. His argument only deals with the

result of that discipline: a lack of desire that is worthyof praise.

22. (C) Assumption (Formal Logic)

Just like in Reading Comprehension, when an authorsays “some people argue that…” you can bet he’sgoing to disagree with them.

The LSAT uses the same formulations over and overagain. That’s one of the huge advantages of preparingwith Kaplan. We teach you to recognize things like thephrase “some people argue…” in this stimulus andknow what to do with them. Here, we know before weeven get to the Keyword “therefore” in the finalsentence that the economist will conclude that largecountries splitting into small countries doesn’tincrease barriers to free trade.

The key to answering this question will be finding thegap between that conclusion and the evidence theeconomist uses to support it. It turns out that there’sonly one piece of evidence, that small countries don’tconsider themselves economically self-sufficient. Well,there’s certainly a gap between a country thinking ofitself as economically self-sufficient and that countryimposing national tariffs or other barriers to free trade,so the economist’s assumption has to fill it. We canparaphrase that assumption in Formal Logic terms likeso: if a country doesn’t think of itself as self-sufficient,then it won’t create barriers to free trade. (C) phrasesthis assumption in its contrapositive form.

(A) The right of countries to split is irrelevant to theeconomist’s argument. He’s only concerned with theeffects of such a split after the fact.

(B) An increased number of countries isn’t the centralconcern of the argument—an increased number ofnational tariffs is. While it’s easy to assume that morecountries means more tariffs, the economist actuallyargues that this assumption is false.

(D) The harm to the world economy caused by tariffsand other barriers to free trade is mentioned asevidence in the argument. Whether or not there is strongevidence for that harm isn’t important, because we haveto take the evidence in the stimulus at face value.

(E) is implied by the fact that small countries don’tthink of themselves as self-sufficient, but it isn’tnecessary to the economist’s conclusion. Theassumption in this question, one that “enables” theconclusion to follow, must be needed.

Section IV: Logical Reasoning

45

Page 48: LSAT_PT 49_Expl_web

23. (D) Flaw (Formal Logic)

Certain flaws can be phrased more than one way—forexample, overlooking an alternative possibility isreally the same thing as confusing necessity andsufficiency.

If only things were really as simple as this counselorclaims, we could all learn to be more accepting ofourselves and others. Unfortunately, the question stemtells us that her argument doesn’t hold water, and ourjob is to figure out why.

The counselor gives us the results of comparingourselves to others. This is a pretty simple cause-and-effect argument, and as such can be phrased asformal logic: if the other people are more able or moresuccessful, we’ll end up disparaging ourselves; if theother people are less able or less successful, we’ll endup being dismissive of others. But this only tells uswhat happens when we compare oneself to others. Asin any Formal Logic statement, we can’t be certainwhat happens if we don’t pull the trigger—if we don’tcompare ourselves to others. Unfor tunately, thecounselor bases her conclusion on exactly that, andtherein lies the flaw in her argument. We know from ourwork with Formal Logic that you can always get theresult of a Formal Logic statement without pulling thetrigger. In this case, that would mean that it’s possibleto become self-disparaging or dismissive of otherswithout comparing oneself to others. The counseloroverlooks that alternative possibility, mistaking onesufficient cause for a result as the necessary cause,as (D) points out.

(A) The results of comparing oneself to both groupswould almost certainly be a combination of the resultsof comparing oneself to each group separately. As longas these results aren’t mutually exclusive (and theyaren’t here), the author’s reasoning allows for thatpossibility, so this isn’t our flaw.

(B) The beneficial effects of making comparisonsbetween oneself and others are outside the scope ofthe argument; the counselor is only concerned withavoiding the detrimental effects of the comparisons.

(C) just doesn’t make much sense. If one is dismissiveof others, one isn’t accepting of them, and if one isself-disparaging, one isn’t self-accepting. Even if theauthor does take this for granted, it’s common sense,not the argument’s flaw.

(E) Although the argument does not acknowledgethose who are neither more nor less successful thanoneself, filling this in as something the author “takesfor granted” does nothing to rectify this flawed

conclusion, as a correct assumption (no matter howabsurd) must.

24. (A) Parallel Reasoning (Flaw, formal logic)

The correct answer to a Parallel Flaw question mustnot only have the same type of conclusion and thesame type of evidence, but also the same type of flaw.

The stimulus deals with a fictional computer company,Compujack, and the salaries of its computerprogrammers. It asserts a simple fact about thoseprogrammers: at least one of them must be paid anexcellent salary. The evidence requires us tounderstand that “most” on the LSAT means at leasthalf, or 50% plus one, and work with the implicationsof that fact.

According to the evidence, most computer programmersreceive excellent salaries, and most of the employeesof Compujack are computer programmers. But there’sno way to know whether or not those two majoritiesoverlap: perhaps the computer programmers atCompujack are a very small fraction of all computerprogrammers, so a fact about “most” computerprogrammers doesn’t necessarily come into play. Butthe author acts like the majorities of those groups dooverlap, and that’s the flaw in his argument. When welook for an answer choice where the majoritiesmentioned in the evidence don’t have to have theoverlap asserted in the conclusion, we find it in (A).

(B) isn’t flawed at all. (B) asserts a potential overlapinstead of a definite one, so its conclusion is correctbased on the evidence.

(C) comes close to being parallel with the stimulus, butfalls short. It has the same flaw, drawing majoritiesfrom two different groups and asserting that theyoverlap, but the conclusion includes a qualification(one of Molly’s classmates who is a gardener) that theconclusion in the stimulus does not.

(D) again has a similar flaw to that in the stimulus, butthe second group in the evidence has twocharacteristics (Molly’s classmates who garden andare women) instead of the single characteristic(computer programmers) used to define the group inthe stimulus.

(E) Like (B), (E) isn’t flawed. This choice picks twomajorities—gardeners with patience and women—outof the same group, Molly’s class. Two majorities of thesame group must overlap, since both of the majoritiesinclude more than half of the group.

PrepTest 49 Explained

46

Page 49: LSAT_PT 49_Expl_web

25. (B) Paradox / EXCEPT

Sometimes it’s easier to reach the correct answer toan EXCEPT question by eliminating answer choicesthat are clearly wrong.

Four of the answer choices to this question will explainthe different average lengths of visits to the museum,but the one we’re looking for won’t do so. Thisdifference stems from the special exhibitions themuseum sometimes offers. When there is not aspecial exhibition, the museum’s attendance is lower,but the patrons tend to spend a longer time at themuseum. We can eliminate any answer choice thataccounts for the difference, so let’s evaluate thechoices:

(A) Visitors who come to see the special exhibitionsdon’t view as many exhibits, so it would make sensethat they don’t spend as much time at the museum. (A)helps to explain the difference, so we can eliminate it.

(B) A plan to extend museum hours that wasn’t evenput into place couldn’t possibly affect how long visitorsstay in the museum. (B) wouldn’t explain thedifference, so it is our correct answer. Let’s quickly seehow the last three choices explain the difference inhow long different visitors spent at the museum:

(C) You can get the prestige of having been somewherejust by walking in and walking out, so visitors just in itfor that purpose don’t need to stay long. (C) helpsexplain the difference.

(D) If admission to special exhibitions has a time limit,then visitors to those exhibits could easily have theirvisits cut short when compared to regular visitors. Wecan eliminate (D).

(E) Without the opportunity to browse, it makes sensethat visitors to special exhibitions wouldn’t spend asmuch time looking at different exhibits as regularpatrons. Eliminate this choice.

Section IV: Logical Reasoning

47

Page 50: LSAT_PT 49_Expl_web

*LSAT is a registered trademark of the Law School Admission Council.

1-800-KAP-TEST | kaptest.com

ÖLL3202A_äLL3202A

Printed in USA ©2006 Kaplan, Inc.